Hand Fractures/Dislocations, Nerves, and Tendon Injuries

Lakukan tugas rumah & ujian kamu dengan baik sekarang menggunakan Quizwiz!

A 3-year-old girl is brought to the office for follow-up because she is unable to flex the interphalangeal joint of the thumb of the dominant right hand. She underwent repair of a laceration of the thenar eminence of the affected hand 8 weeks ago. Operative exploration shows a 3-cm gap of the flexor pollicis longus in Zone III. Reconstruction with a palmaris longus graft is planned. Which of the following is the most appropriate postoperative management? A) Complete immobilization for 4 weeks B) Removable dorsal-block splint; passive and active-assist flexion C) Removable dorsal-block splint; passive flexion D) Removable dorsal-block splint; passive, active-assist, active flexion E) No immobilization

A Early motion protocols are standard for adult tendon repairs but are not generally suitable for very young children due to poor compliance. Children have a remarkable ability to regain motion after tendon injury, especially for repairs or grafts that are outside of Zone II. In this vignette, the reconstruction was in Zone III, and the prognosis for regaining full motion even after a month of immobilization is excellent. Moreover, it is highly unlikely that a child of this age will predictably comply with splint wear and motion restrictions for the duration of tendon healing. Thus, the risk of early rupture outweighs the risk of stiffness. There are advocates of successful early motion protocols in children with Zone II tendon repairs, but most studies have failed to demonstrate an appreciable benefit of such a practice.

A 42-year-old woman comes to the emergency department after sustaining a deep laceration to the dorsal aspect of the right forearm with broken glass. She reports pain on attempted extension of the middle and ring fingers of the right hand. On physical examination, the patient cannot extend her fingers beyond the neutral position with her right hand held flat on a table. She is able to extend all digits completely at the interphalangeal joints in all positions of the hand. Which of the following is the most likely cause of these findings? A) Extrinsic extensor tendons are cut completely with intact intrinsic muscles B) Extrinsic extensor tendons can extend the digits despite the laceration injury C) Juncturae tendinum are extending the digits D) The patient has an accessory extensor tendon E) The patient has partial extensor tendon lacerations only

A Extension of the interphalangeal joints (IP) and extension of the metacarpophalangeal joints (MCP) to the neutral position after an injury to the forearm extensor tendons is possible through the action of the lumbricals and dorsal interossei (intrinsic hand muscles). The extrinsic extensor tendons alone are responsible for extending the metacarpophalnageal joints beyond neutral. The role of the juncturae tendinum is to limit the independent extension of the ulnar three digits. The juncturae tendinum will allow for some finger extension with an isolated single tendon injury at the wrist or dorsal hand but would not maintain extension in all fingers. No meaningful accessory extensor tendon exists. A patient with partial extrinsic tendon lacerations may present with pain on attempted extension but extension beyond neutral would be intact.

A 38-year-old woman comes for evaluation 7 weeks after undergoing a Zone II flexor tendon repair of the left long finger. She reports feeling a "pop" at home and is now unable to flex the finger at the proximal or distal interphalangeal joints. On examination today, the finger is swollen with moderate stiffness. Flexor tendon rupture is suspected. During operative exploration, ruptures of the flexor digitorum superficialis (FDS) and flexor digitorum profundus tendons are noted. There is a 1.5-cm gap of the profundus tendon and scarring at the A2 pulley. Which of the following is the most appropriate next step in management? A) Excision of the flexor tendons with implantation of a passive silicone rod prosthesis B) Fractional lengthening of the tendon in the forearm and revision of primary repair C) Single-stage reconstruction with implantation of an active silicone rod prosthesis D) Single-stage tendon reconstruction with palmaris graft E) Tendon transfer from the ring finger FDS

A Flexor tendon reconstruction in Zone II was originally described by Bassett and Carroll in 1963 and refined by Hunter in 1971. In the first stage, a Dacron-reinforced silicone rod is implanted after excision of the native tendons. It is secured distally to the flexor digitorum profundus stump or directly to the distal phalanx. Proximally, the rod is placed adjacent to the motor tendon but not secured. At this time, pulley reconstruction with tendon or retinacular grafts can be performed as indicated. This allows formation of a pseudosheath around the rod. Once the soft tissue has healed and the patient has regained maximum passive range of motion through therapy, the second-stage tendon grafting is performed. The most commonly reported time frame is 3 months but depends on soft-tissue stability. The most important management decision is to determine if primary repair is possible. If not, one must then decide between single-stage and two-stage tendon reconstruction. The criteria for single-stage reconstruction include a finger with adequate passive motion of all joints, soft tissues with minimal scarring, functional tendon sheath and pulley system, a neurovascularly intact digit, and a compliant patient. If these criteria are not met, the patient should be treated with a staged reconstruction. In this patient, primary repair is not likely with a gap >1 cm 7 weeks after the initial repair. Myostatic contraction of the proximal stump would be expected. In addition, this patient's digit has significant internal scarring and poor passive motion— all factors that preclude a single-stage repair. Fractional lengthening might be considered to allow primary repair if the tendon sheath was better quality. If a single-stage repair were indicated, one may consider a flexor digitorum superficialis transfer from an adjacent digit as a motor for the transfer. This requires only one tendon anastomosis, and studies have shown decreased adhesion formation with intrasynovial tendon grafts. However, tensioning of the transfer can be more difficult than traditional tendon grafting. Patients who are unable to tolerate a second-stage procedure can be considered for the implantation of an active silicone rod prosthesis. This device is designed to have both a distal anastomosis to the bone and a proximal anastomosis to the motor muscle-tendon unit with integrated sutures or a loop. This would require meeting the criteria for single-stage reconstruction. Active implants were originally designed for use in two-stage reconstruction, but no studies exist that compare active with passive silicone rods or show any benefit versus a passive implant in a staged reconstruction.

A 30-year-old man is evaluated after sustaining a laceration of the left ulnar nerve at the level of the mid humerus. Exploration and direct repair of the ulnar nerve are performed immediately. Which of the following interventions is most likely to result in optimal recovery of hand function? A) Anterior interosseus nerve transfer to the ulnar nerve B) Dynamic splinting C) Electrical stimulation of the ulnar nerve D) Flexor digitorum superficialis tendon transfer to the A2 pulley E) Wrist extensor tendon transfer to the lumbricals

A High ulnar nerve injuries are plagued by poor recovery of intrinsic function even when the nerves are repaired under optimal conditions. To maximize recovery and restore intrinsic function in the hand, one of the most effective interventions is transfer of the motor branch of the anterior interosseus nerve to the motor branch of the ulnar nerve at the wrist. Recently, this has been shown to also be useful as a "babysitter" transfer, potentially retaining motor endplates in the small muscles of the hand while a primarily repaired ulnar nerve regenerates. Dynamic splinting has played a role in radial palsies and is frequently employed to overcome joint contractures that are not fixed (with a hard endpoint). This would not assist in hand recovery in this setting. Electrical stimulation performed by a hand therapist helps in retraining innervated muscles but will not improve function in the denervated small muscles of the hand. Tendon transfers have been the mainstay of functional restoration when nerve repair is not possible and/or the timeframe that has elapsed is great enough to make successful reinnervation of the target muscles unlikely. Both the wrist extensor-to-lumbrical transfer and the flexor digitorum superficialis transfer have been used to correct the "clawing" associated with ulnar palsies.

A 17-year-old boy comes to the office because of ongoing pain of the right hand after he punched a wall 5 days ago. Physical examination demonstrates tenderness of the fifth carpometacarpal joint. Posteroanterior, oblique, and lateral x-ray studies taken at an urgent care facility were read as negative by the radiologist. Which of the following additional radiographic views is most likely to help confirm this patient's diagnosis? A) Anteroposterior with 30 degrees of pronation from full supination B) Carpal tunnel view with wrist in full extension C) Clenched fist lateral in neutral forearm position D) Lateral with 15 degrees of supination from neutral forearm position E) Posteroanterior with 45 degrees of supination from full pronation

A Injuries to the fifth carpometacarpal joint, including subluxation, dislocation, and fracture-dislocation, are often missed with standard two-view and three-view hand images. Two views have been suggested to help detect this subtle injury: Anteroposterior view with forearm pronated 30 degrees from full supination. This view shows more clearly the profile of the articulation between the hamate and fifth metacarpal base. (This is similar to the "reverse oblique" view, which is typically done in 45 degrees of pronation and might also be useful.) Lateral with 30 degrees of pronation. This view is especially helpful for detecting subluxation of the metacarpal dorsally off of the hamate. In some cases, CT scan may be warranted if plain films are inconclusive.

A 14-year-old boy sustains a laceration of the distal forearm. Physical examination and wound exploration suggest flexor carpi ulnaris tendon and ulnar nerve involvement. The patient undergoes immediate microsurgical nerve repair with a nerve graft. Which of the following factors is most likely to predict a satisfactory outcome in this patient? A) Age B) Gender C) Immediate repair D) Ulnar nerve involvement E) Use of nerve graft

A Multiple studies have evaluated outcomes of median and ulnar nerve repair after transection injury. A meta-analysis by Ruijs et al. confirmed that younger age, specifically under 16 years old, was associated with the highest chance of satisfactory recovery of motor function in both median and ulnar nerve injuries. Patient gender was not found to be a statistically significant factor in outcome. In the same analysis, median motor nerve injuries were found to have a better chance of recovery than ulnar motor nerve injuries. Timing influenced outcome, with delay of repair adversely affecting prognosis, and although the ideal window for repair was not able to be defined by this review, there is some evidence that immediate repair may result in worse outcomes. The use of nerve grafts did not significantly predict motor recovery in these injuries.

A 43-year-old woman is evaluated for intrinsic wasting and paresthesias of the little and ring fingers. Which of the following is the most likely site of nerve entrapment? A) Anconeus epitrochlearis B) Arcade of Frohse C) Lacertus fibrosis D) Ligament of Struthers E) Pronator teres

A Nerve entrapment is caused by the anconeous epitrochlearis. The scenario describes both motor and sensory signs and symptoms attributable to ulnar nerve compression. Ulnar nerve entrapment at the elbow is the second most common nerve entrapment neuropathy in the upper limb other than carpal tunnel syndrome in young adults. The most common cause is the ligament of Osborne. The anconeus epitrochlearis muscle, which is a congenital accessory muscle, arises from the medial epicondyle of the humerus and inserts at the olecranon process of the ulna. It can be found in normal elbows with an incidence of between 4 and 34%. It has been associated with ulnar nerve compression at the elbow. The arcade of Frohse has been associated with posterior interosseous nerve entrapment. The lacertus fibrosis, pronator teres, and ligament of Struthers have been associated with median nerve compression. Note that the arcade of Struthers may be associated with ulnar nerve compression, but this option is not provided.

A 42-year-old man with carpal tunnel syndrome is evaluated for symptoms that are progressively worsening despite conservative management. Surgical release using an open, short scar technique is planned. Which of the following is the most accurate statement when comparing this technique with endoscopic release? A) Both techniques are equivalent in long-term symptom relief and recovery B) Endoscopic release is far superior because of superior long-term symptom relief C) Endoscopic release is only used for bilateral cases D) Open, short scar technique requires regional block, whereas endoscopic release does not E) Open technique has a higher association with recurrent median nerve injury

A Open, short incision and endoscopic carpal tunnel release are equivalent in long-term symptom relief and recovery. Carpal tunnel syndrome is a condition caused by compression of the median nerve at the wrist. It is characterized by pain and numbness of the fingers within the median nerve distribution: the thumb, index, and long fingers, as well as the radial aspect of the ring fingers. With progressive compression, thenar atrophy can occur as well as weakness of thumb opposition. Conservative treatment includes splinting, avoidance of repetitive activities or positions that elicit symptoms, and occasionally steroid injection. With progression of symptoms, surgical release is indicated. Open release provides transcutaneous access to the transverse carpal ligament. Traditionally, a long incision had been used extending from the proximal palm across the wrist and onto the proximal forearm. The incision has become progressively shorter, such that most surgeons employ a short scar confined to the proximal palm. Endoscopic release uses two small incisions for port access and provides transection of the transverse carpal ligament without division of the palmar aponeurosis. Debate has existed regarding the superiority or inferiority of one technique over the others. Clearly, the endoscopic technique causes less pain and less alteration in early grip strength, when compared with the more classic, longer incision open techniques. However, when specifically comparing the open, limited scar technique to the endoscopic technique, studies have shown essentially the same outcome data regarding strength, return to work, symptom relief, and reoperation. Virtually all studies have shown that open and endoscopic release have the same long-term symptom relief, measured at multiple points in time up to one year. The open technique is often thought to be associated with a lower association with recurrent median nerve injury. Bilaterality does not preclude open or endoscopic release. Both techniques can be done during local or regional anesthesia.

A 35-year-old woman is evaluated 2 months after repair of volar lacerations to the dominant ring and long fingers at Zone III in both digits. Physical examination shows both fingers have no active flexion. There is normal passive motion and normal sensation at the fingertips of the affected digits. Surgical exploration shows transsection of the tendons at both levels and 2-cm segmental tendon loss, but no tendon sheath scarring. Which of the following is the most appropriate management? A) One-stage tendon grafting B) Primary tendon repair C) Primary tendon repair with z-lengthening D) Superficialis to profundus tendon transfer E) Two-stage tendon grafting

A Several conditions must be met for single-stage tendon grafting to be successful. These include a hand and finger that have good passive motion, a well-healed wound with minimal scarring, and a digit that has intact nerves and arteries. This is a Boyes grade 1 injury. If the grade of injury is greater than 1, two-stage grafting should be considered with implantation of a silicone rod and additional treatment to manage the other conditions to increase motion and function, such as joint release or reconstruction for loss of motion, nerve repair/reconstruction, and pulley reconstruction. A primary repair is not going to be possible in this instance. Two-stage tendon grafting should be considered but a single stage graft can have excellent results with only one procedure. A tendon transfer is not appropriate for this situation.

A 6-year-old boy presents with a supracondylar fracture sustained during a fall on an outstretched hand. A splint with the elbow flexed less than 90 degrees is placed. The patient is screaming in pain. Examination shows the affected hand has a 3-second capillary refill. Which of the following is the most appropriate next step in management? A) Closed reduction B) Continued observation and application of ice packs C) Elevation of the arm D) Exploration of the brachial artery E) Replacement of the current splint with an elbow extension splint

A Supracondylar fractures are one of the most common traumatic fractures seen in children. It occurs most commonly in children 5 to 7 years of age with similar male and female incidence. The mechanism is usually from a fall onto an outstretched hand. The fracture can lead to severe forearm edema, then ischemia leading to Volkmann's contracture. Immobilization would be long arm casting with the elbow flexed at less than 90 degrees. Arm elevation would decrease tissue perfusion and would therefore be contraindicated. Immediate bedside closed reduction by gentle traction and elbow flexion to 20 to 40 degrees would be indicated in this case as a next step. If the closed reduction is unsuccessful or ischemia persists after reduction or recurs, urgent operative closed reduction with percutaneous pinning is required. Pins are placed to prevent recurrence. Brachial artery exploration could be required if ischemia has not resolved even after successful reduction, but not initially.

In a transhumeral amputee, targeted muscle reinnervation can be utilized to improve control in a myoelectric prosthesis. Which of the following nerve transfers can be performed to provide intuitive prosthetic control for hand closure? A) Median nerve to short head of biceps B) Musculocutaneous nerve to long head of biceps C) Radial nerve to lateral head of triceps D) Radial nerve to long head of triceps E) Ulnar nerve to lateral head of triceps

A Targeted muscle reinnervation (TMR) utilizes a set of nerve transfers in order to allow intuitive prosthetic control for upper extremity amputees. Functioning nerves that no longer have their distal muscle target can be transferred to intact proximal muscles and generate a novel electrical signal that can be picked up by a myoelectric prosthesis. Another benefit of TMR is the potential to prevent or treat painful neuromas. In the case of a transhumeral amputee, elbow flexion myoelectric prosthetic control is maintained by preserving musculocutaneous innervation to the long head of the biceps muscle. The distal remnant of the median nerve is transferred to the motor nerve of the biceps short head to create a signal for prosthesis hand closure. Elbow extension signals are maintained with radial innervation of the long head of the triceps. Signals for prosthesis hand opening are created with transfer of the distal radial nerve to the motor nerve of the triceps lateral head.

A 47-year-old man is brought to the emergency department after sustaining a stab wound injury to the left shoulder. Physical examination shows isolated loss of deltoid function. The injured nerve is supplied by which of the following nerve roots? A) C5 through C6 B) C6 through C7 C) C7 only D) C7 through T1 E) T1 only

A The deltoid muscle receives motor innervation from the axillary nerve. The axillary nerve receives its contributions from C5 and C6 roots. These roots come together to form the superior trunk, which splits into anterior and posterior divisions. The axons heading to the axillary nerve travel in the posterior division, which joins the other posterior divisions from middle and inferior trunks to form the posterior cord. The axillary nerve arises from the posterior cord and travels laterally to innervate the deltoid muscle. C7 is the primary innervation to the latissimus dorsi and triceps, and contributes to digital extension as well. C8 and T1 primarily serve the hand, providing intrinsic muscle innervation.

A 54-year-old woman comes to the office because of a 6-year history of weakness and numbness of the left hand. Physical examination shows decreased sensation in the thumb, index, long, and ring fingers. No other sensory abnormalities are noted. Examination of which of the following muscles is most likely to confirm a diagnosis? A ) Abductor pollicis brevis B ) Adductor pollicis C ) First dorsal interosseous D ) Flexor digiti minimi E ) Flexor pollicis brevis

A The examination of the patient described suggests an injury or compression neuropathy of the median nerve. The only intrinsic muscle innervated by the median nerve (recurrent branch) that can be reliably tested separately from the ulnar intrinsic muscles is the abductor pollicis brevis. The adductor pollicis, first dorsal interosseous, and flexor digiti minimi are all completely innervated by the ulnar nerve. The flexor pollicis brevis muscle has dual innervation from both the ulnar (deep head) and median (superficial head) nerves.

A patient with severe traumatic brachial plexus root avulsion injury is scheduled to undergo functioning free muscle transfer for simultaneous restoration of both elbow flexion and finger flexion, in addition to other reconstructive procedures. Use of which of the following muscles is most appropriate for this purpose? A) Gracilis B) Pectoralis major C) Rectus abdominus D) Serratus anterior E) Trapezius

A The gracilis muscle is the most commonly described muscle for use as a free functioning muscle in reconstruction of upper extremity function following brachial plexus injury. Common options for use in these reconstructions include the gracilis, latissimus dorsi, rectus femoris, and vastus lateralis. The gracilis muscle has good excursion, size, and length, but does lack strength compared with some other muscle options. The rectus abdominis, serratus anterior, and trapezius muscles have not been described for free functioning muscle transfer in the upper extremity. Though the pectoralis major muscle was described as a free functioning muscle transfer by Manktelow and McKee in 1978, it has not been a commonly used muscle.

A 34-year-old machinist undergoes repair of the flexor tendon of the index finger as the result of a work-related injury. Which of the following is the main rationale for performing early motion exercises after surgical repair? A) Decrease adhesions B) Decrease postoperative pain C) Improve strength of repair D) Increase synovial fluid flow E) Prevent rupture

A The main rationale for performing early motion exercises is to decrease adhesion formation. During the early phases of tendon healing, large amounts of collagen are deposited and form early scarring. Although this scarring is crucial for the healing of the repaired tendon, scarring will also occur in the tendon sheath and, if allowed to progress, can lead to stiffness in the involved digit. Early passive- and active-motion protocols assist in breaking apart early scarring of the tendon to the surrounding sheath. Early motion has not shown to increase the final strength of the repair and is more likely to cause rupture. Although synovial fluid flow might be increased with tendon excursion, it does not improve outcomes. Postoperative pain is likely increased with therapy.

A 29-year-old man is brought to the emergency department because of a deep laceration of the medial right arm at the elbow. Examination shows complete loss of ulnar nerve function to the right hand. During exploration in the operating room, complete transection of the ulnar nerve at the elbow with a 3-cm nerve gap after debridement is noted. Which of the following procedures is most likely to result in the earliest recovery of intrinsic muscle function in this patient's hand? A) Anterior interosseous nerve transfer B) Cable grafting with nerve allografting C) Cable grafting with sural nerve grafting D) Primary repair with the elbow in flexion E) Ulnar nerve transposition and primary repair

A The most appropriate answer is transfer of the anterior interosseous nerve to the motor branch of the ulnar nerve in the distal forearm. Ulnar nerve injuries are especially debilitating with loss of grip and claw hand deformity. High ulnar nerve injuries are considered to be anything proximal to the innervation of the flexor carpi ulnaris and flexor digitorum profundus muscles at or near the elbow. This includes the elbow, upper arm, and brachial plexus. The prognosis for recovery of intrinsic hand function is poor in high ulnar nerve injuries 30 to 35 cm proximal to the hypothenar eminence. This is due to the length of time required for nerve regeneration, and the motor endplate degeneration that occurs during this time. High ulnar nerve injuries will exceed the approximately 18-month window for regeneration in order to achieve meaningful muscle recovery. The theory behind nerve transfer surgery is to take an expendable donor nerve and use the fascicles to restore function to a more critically injured nerve. The anastamosis for an AIN-to-ulnar nerve transfer is 8 to 10 cm proximal to the wrist crease and greatly decreases the amount of distance and therefore time required for reinnervation of intrinsic hand muscles. Although the anterior interosseous nerve contains 75% of the axons of the deep motor branch of the ulnar nerve, meaningful recovery of intrinsic muscle function can be expected. The anastamosis for the transfer can be done in an end:end or end:side technique. End-to-side nerve transfer is indicated in partial nerve injuries or lower nerve injuries where primary repair of the injured nerve is possible and one can expect some contribution of nerve fibers from the native nerve. Primary repair of any nerve injury under tension or that requires extreme joint flexion is not indicated. This results in tension at the repair site, internal scarring, possible flexion contracture, and a poor result. Anterior transposition of the ulnar nerve may be performed in some instances but is reported to gain only 1 cm of length from the proximal nerve. This is not enough to bridge the gap in this scenario. Any attempt at primary repair of a nerve injury with a 3-cm gap would require nerve grafting. In a mixed nerve, multiple cables of nerve graft are recommended in an attempt to topographically reconnect the sensory and motor fascicular bundles. The current gold standard for nerve repair in adults is autograft. The most common donor nerves are the sural nerve and medial antebrachial cutaneous nerve. Processed nerve allograft has become a viable alternative to autograft nerve. The allograft nerve is processed and decellularized but maintains the microstructure of the nerve tissue including the fascicular anatomy and microvasculature. The allograft is rapidly revascularized without the donor site morbidity associated with autograft. The RANGER study has demonstrated S3 and M4 or above recovery in 86% of repairs using allograft nerve in gaps up to 5 cm. This question specifically asks about the most rapid recovery of motor function, which should occur with a nerve transfer.

A 57-year-old woman comes to the office because of burning pain and stiffness of the right hand 8 weeks after closed treatment of a distal radius fracture. The patient reports that she has had difficulty sleeping and continues to have discomfort despite taking narcotics. On physical examination, the hand is shiny, swollen, and warm, and finger range of motion is decreased. There is hypersensitivity to light touch. X-ray studies show good alignment of the fracture. Electrodiagnostic testing shows no abnormalities. Bone scan shows increased periarticular uptake. Which of the following is the most appropriate diagnosis? A) Complex regional pain syndrome B) Factitious disorder C) Midpalmar space abscess D) Opioid addiction E) Pain catastrophizing

A The most appropriate diagnosis is reflex sympathetic dystrophy, or complex regional pain syndrome (CRPS) type I. This patient exhibits symptoms consistent with CRPS, which is a form of severe neuropathic pain. The diagnosis of CRPS involves history, physical examination, and diagnostic testing. In addition to pain out of proportion, other features must be present. These can include changes in blood flow, altered temperature perceptions, sudomotor activity, edema, and pigmentation changes. Although no specific test is pathognomonic, triple-phase bone scans are helpful in adding credence to the diagnosis. First- and second-phase bone scans may show asymmetric flow and autonomic dysfunction, while the third phase demonstrates increased periarticular uptake in multiple joints of the affected extremity. CRPS is divided into two types. Type I occurs without identifiable nerve involvement (also known as reflex sympathetic dystrophy), and Type II has identifiable nerve involvement (causalgia). It is more common in smokers and in women. Pain in CRPS can be either sympathetically mediated or sympathetically independent. This condition is characterized by persistent pain, cold intolerance, autonomic dysfunction, and trophic changes. Patients may show swelling, stiffness, difficulty sleeping, and persistent pain out of proportion to the normal postoperative/post-injury course that may be incompletely relieved by narcotics. A variety of treatment modalities have been employed in addressing CRPS. These range from therapy modalities such as range of motion, stress loading, and desensitization to pharmacologic interventions with anticonvulsants or antidepressants. Stellate ganglion blocks or autonomic nerve blocks may be helpful in sympathetically mediated pain, and nerve stimulation (either transcutaneous or at the spinal cord level) can also be employed. Often multiple modalities are used concurrently and in sequence. Peripheral nerve decompression may be helpful in resolving symptoms related to CRPS type II. Factitious disorder can occur when there is potential for secondary gain, but would not present with physiological symptoms. Although opioid addiction can be a source of pain complaints in an attempt to acquire additional narcotics, the patient exhibits physiological changes that are unable to be mimicked. Pain catastrophizing is a maladaptive behavioral response to pain that can be a risk factor for prolonged pain after trauma. A midpalmar space abscess would be unlikely after a closed distal radius fracture. It would also not be likely to have trophic skin changes or changes in a bone scan as seen in this patient.

A 32-year-old woman comes to the office for evaluation because of numbness of the left little finger 3 months after undergoing repair of a laceration of the left wrist sustained during an unsuccessful suicide attempt. Physical examination shows a healed laceration with a dysesthetic scar at the proximal wrist crease. A strong Tinel sign is present at the repair site. There is complete sensory loss of the little finger and no evidence of clawing. Motor function is intact. Wartenberg sign is absent. Two-point discrimination is greater than 15 mm. Which of the following is the most appropriate next step? A) Microdissect the neuroma and identify motor fascicles with electrostimulation B) Microdissect the neuroma and sural nerve graft fascicles C) Resect the neuroma and direct repair with transposition D) Resect the neuroma and repair with sural nerve grafts

A The patient described has an ulnar neuroma-in-continuity with intact motor function and no sensory regeneration. Mackinnon has described an electrostimulation technique where the proximal motor fibers are identified using nerve stimulation. Resection of the neuroma, with or without transposition, is not appropriate because it would cause damage to intact nerve fascicles. Microdissection without nerve stimulation would also cause damage to intact fascicles.

A 20-year-old man comes for evaluation 9 months after sustaining a stab wound to the left proximal upper arm. He did not seek medical attention at the time of the injury. Physical examination shows that he is unable to flex the left elbow with the forearm supinated. He is insensate to the lateral aspect of the upper arm and forearm. In addition to the ulnar nerve transfer to the biceps, which of the following nerve transfers is most appropriate to address this patient's motor deficit? A) Median nerve to brachialis B) Musculocutaneous nerve to brachioradialis C) Posterior interosseous nerve to triceps D) Radial nerve to pronator teres E) Ulnar nerve to flexor carpi radialis

A The patient described has sustained a laceration of the musculocutaneous nerve. With this injury, the patient would be able to flex the elbow with the forearm in a pronated position using the brachioradialis, which is innervated by the radial nerve. Due to the proximal level and amount of time that has passed since the injury, repair with grafting of the musculocutaneous nerve may not be advisable as the axons may not reach the motor end plates of the biceps and brachialis muscles before degeneration. Use of fascicles from the median nerve, ulnar nerve, and both nerves has been described to restore elbow flexion. Fascicles are transferred distally in the upper arm directly to the nerve branch to the brachialis and/or biceps muscle. Distal coaptation allows donor axons to reach the target muscles more rapidly, and is more appropriate for this patient whose status is nearly 1 year post injury. A fascicle from the ulnar nerve was transferred to the nerve to the biceps (blue dot) and a fascicle from the median nerve was transferred to the nerve to the brachialis (green dot).

Following central slip injury, volar subluxation of the lateral bands can lead to which of the following deformities? A) Boutonnière B) Lumbrical plus C) Mallet D) Quadriga E) Swan-neck

A The triangular ligament stabilizes the lateral bands dorsally, thereby preventing volar subluxation of the lateral bands to the proximal interphalangeal (PIP) joint rotation of axis, and the boutonnière deformity. The swan-neck deformity occurs when the lateral bands sublux dorsal to the PIP joint rotation of axis. This is prevented by the transverse retinacular ligament, which acts to prevent dorsal migration of the lateral bands at the PIP joint. Neither the mallet, quadriga, or lumbrical plus deformities are caused by volar subluxation of the lateral bands.

A 32-year-old man presents to the emergency department for evaluation of a laceration of the right wrist sustained when he punched a glass window 1 hour ago. Physical examination shows a 2-cm transverse laceration of the volar ulnar wrist crease. Wound exploration shows complete laceration of the ulnar nerve. On physical examination of motor function, LOSS of which of the following functions is most likely in this patient? A) Adduction of the thumb carpometacarpal joint B) Extension of the metacarpophalangeal joint of the ring and small fingers C) Extension of the thumb interphalangeal joint D) Flexion of the interphalangeal joint of the index and middle fingers E) Flexion of the interphalangeal joint of the ring and small fingers

A The ulnar nerve is the terminal branch of the medial cord of the brachial plexus. It enters the forearm between the two heads of the flexor carpi ulnaris (FCU). In the forearm, the ulnar nerve innervates the FCU and flexor digitorum profundus of the small and ring fingers. It courses distally under the FCU to enter Guyon's canal at the wrist. The dorsal cutaneous nerve, which gives sensation to the dorsoulnar hand, arises approximately 5 to 7 cm proximal to the ulnar styloid. In Guyon's canal, the ulnar nerve splits into a deep motor and a superficial sensory branch. The deep motor branch innervates the hypothenar muscles (abductor digiti minimi, opponens digiti minimi, and flexor digiti minimi), as well as the lumbricals to the ring/small fingers, dorsal and palmar interossei, flexor pollicis brevis (deep head), palmaris brevis, and adductor pollicis. The superficial sensory branch in the palm innervates the small finger and the ulnar aspect of the ring finger. Adduction of the thumb is controlled through activation of the ulnar-innervated adductor pollicis muscle. In the small and ring fingers, extension of the metacarpophalangeal (MCP) joint is performed through activation of the radially innervated extensor digitorum communis and extensor digiti minimi muscles. In the index, middle, ring, and small fingers, flexion of the proximal interphalangeal (PIP) joint is performed through activation of the median-innervated flexor digitorum superficialis. While flexion of the ring and small finger distal interphalangeal (DIP) joints is produced by the ulnar-nerve innervated FDP tendons to the ring and small finger, the ulnar nerve provides branches to this muscle proximal to this patient's injury. Extension of the thumb MCP joint is via the radial nerve innervated extensor pollicis brevis muscle.

A 16-year-old boy is brought to the office after "jamming" the right long finger of the dominant hand while playing football. Upon active extension, the patient exhibits an extension lag of 40 degrees at the proximal interphalangeal (PIP) joint, and hyperextension at the distal interphalangeal (DIP) joint. Which of the following is the most likely diagnosis? A) Central slip disruption B) Flexor digitorum profundus avulsion C) PIP volar plate tear D) Sagittal band rupture E) Swan neck deformity

A This is the basis of the Elson test for central slip disruption of the extensor mechanism of the finger. When the proximal interphalangeal (PIP) joint is maximally passively flexed, the central slip is normally pulled distally, resulting in slack in the terminal tendon. Injury to the central slip eliminates this slack through the lateral band and allows extensor tension to be generated at the distal interphalangeal (DIP) joint. Thus, with central slip injury, the DIP joint can be actively extended with maximal PIP flexion. Swan neck deformity results from terminal extensor tendon disruption and total inability to extend the DIP joint independent of PIP position. Flexor digitorum profundus (FDP) avulsion results in inability to flex the DIP joint. In the Elson test, DIP flexion is always possible. PIP volar plate injury may result in jamming of the volar plate within the PIP joint and paradoxical inability to flex (extension contracture) at the PIP joint. There will also be hyperextension PIP joint pain and laxity. Sagittal band disruption results in inability to actively extend at the metacarpophalangeal joint, but the finger can often maintain extension if passively placed in this position.

A 48-year-old woman comes to the office because of burning pain and stiffness in the right hand 6 weeks after treatment of a distal radius fracture. She says she has had difficulty sleeping and that she has discomfort despite taking narcotics. Physical examination shows a shiny appearance of the right hand, decreased range of motion of the fingers, and hypersensitivity to light touch. X-ray studies show good alignment of the fracture. Which of the following tests is the most appropriate to evaluate this patient's condition? A) Bone scan B) CT scan C) Digital subtraction angiography D) Lymphoscintigraphy E) Ultrasonography

A This patient exhibits symptoms of complex regional pain syndrome (CRPS). The persistence of physiological changes after surgery or injury can lead to debilitating consequences. This condition is characterized by persistent pain, cold intolerance, autonomic dysfunction, and trophic changes. Patients may show swelling, stiffness, difficulty sleeping, and persistent pain out of proportion to the normal postoperative course that may be relieved incompletely by narcotics. CRPS is a clinical diagnosis without a single definitive test, and is divided into two types: type I, which occurs without identifiable nerve involvement (also known as reflex sympathetic dystrophy); and type II, which has identifiable nerve involvement (causalgia). It is more common in people who smoke and in women. Pain in CRPS can be either sympathetically mediated or sympathetically independent The diagnosis of CRPS involves history, physical examination, and diagnostic testing. Although no specific test is pathognomonic, triple-phase bone scans are helpful in adding credence to the diagnosis. First- and second-phase bone scans may show asymmetric flow and autonomic dysfunction, while the third phase demonstrates increased periarticular uptake in multiple joints of the affected extremity. A variety of treatment modalities have been employed in addressing CRPS. These range from therapy modalities such as range of motion, stress loading, and desensitization to pharmacologic interventions with anticonvulsants or antidepressants. Stellate ganglion blocks or autonomic nerve blocks may be helpful in sympathetically mediated pain, and nerve stimulation (either transcutaneous or at the spinal cord level) can also be employed. Often, multiple modalities are used concurrently and in sequence. Peripheral nerve decompression may be helpful in resolving symptoms related to CRPS type II. CT scans can be used to assess bony alignment in fractures and are helpful in the evaluation of articular anatomy. Digital subtraction angiography is useful for evaluation of vasculature and circulation Lymphoscintigraphy is used to analyze lymphatic drainage in cases of lymphedema. Ultrasonography can be performed to assess venous outflow and look for deep venous thrombosis.

A 39-year-old man is referred to the office 4 months after repair of a zone II flexor tendon involving both the flexor digitorum superficialis and flexor digitorum profundus tendons to the right long finger. He still has poor range of motion of his long finger. Physical examination shows 45 degrees of active range of motion at the proximal interphalangeal (PIP) joint and 25 degrees of active range of motion at the distal interphalangeal (DIP) joint. Active and passive ranges of motion are equal. Which of the following is the most appropriate next step in management? A) Continued hand therapy to improve passive range of motion B) Flexor tenolysis C) PIP arthrodesis D) Two-stage flexor tendon reconstruction E) Observation to allow for scar remodeling

A This patient has adhesions after flexor tendon injury and repair. In this scenario, the recommended course of action is to continue hand therapy to improve passive range of motion. A successful functional outcome following tendon injury depends on supple joints with full passive range of motion and tendon gliding. The ultimate goal would be to perform flexor tenolysis; however, the indications are clear that the patient must have minimal soft-tissue edema, minimal scarring, and full or near-full passive range of motion. Active tendon range of motion depends on the flexor digitorum superficialis and flexor digitorum profundus gliding within the flexor tendon sheath. Flexor tendon adhesions are a potential complication any time the flexor tendon sheath is violated, as a result of either surgery or trauma. The literature shows a reoperation rate of 6% after flexor tendon repair and an adhesion rate of 4%. This patient is 4 months post-surgery and reports compliance with supervised hand therapy. Despite this, he has poor active and passive range of motion. Although most authors recommend waiting at least 3 to 6 months before attempting tenolysis, this patient is unlikely to improve with observation alone. Tenolysis is a technically demanding procedure, and all patients must be counseled preoperatively that complications such as neurovascular injury, injury to the pulley system, and tendon rupture are possible outcomes. In cases of tendon rupture or patients requiring pulley reconstruction at the time of tenolysis, two-stage tendon reconstruction with implantation of a silicone rod is indicated. However, this would be a salvage procedure only and not first-line treatment. Proximal interphalangeal (PIP) joint arthrodesis would be limited to a salvage procedure in patients who are unable to undergo tendon repair or reconstruction.

A 24-year-old man comes to the office because of a brachial plexus injury sustained in a motorcycle accident. Nerve transfer to the biceps for restoration of elbow flexion is planned. Which of the following fascicles or nerves is the most appropriate donor for the transfer? A) Distal spinal accessory nerve B) Flexor carpi ulnaris fascicle of the ulnar nerve C) Medial pectoral nerve D) Palmaris longus fascicle of the median nerve E) Thoracodorsal nerve

B Adult upper trunk brachial plexus injuries result in significant disability. Several surgical treatment strategies exist, including nerve grafting, nerve transfers, and a combination of both approaches. The flexor carpi ulnaris (FCU) fascicle of the ulnar nerve to biceps transfer was first described by Oberlin et al in 1994. Generally, the donor nerve with the largest caliber and the greatest number of motor axons should be used for elbow flexion. The other suggested nerve transfer options are also possibilities, but are not as preferable as the FCU fascicle transfer. MacKinnon has advocated transfer of the FCU fascicle of the ulnar nerve to the biceps and FCR fascicle of the median nerve to the brachialis to maximize recovery of elbow flexion.

The Jones transfer for radial nerve palsy, specifically flexor carpi ulnaris to extensor digitorum communis III-V, is most likely to have which of the following significant disadvantages? A) Difficulty of dissection B) Inappropriate excursion of donor tendon C) Indirect line of pull requiring pulley creation D) Poor synergy E) Unacceptable postoperative rupture rate

B Although the Jones transfers were practiced for years, there were significant disadvantages that led to them falling out of favor. Among the disadvantages are loss of flexor carpi ulnaris (FCU) as an important ulnar wrist stabilizer and weakness in flexion/ulnar deviation, which is a very important wrist motion. Additionally, the short excursion of FCU is inadequate to fully extend the fingers when transferred to extensor digitorum communis (EDC). Subsequent modifications to the Jones technique by Boyes and others found better alternatives to the use of FCU. Standard Boyes transfers are: PT to ECRL and ECRB FCR to EPL and ABL FDS-III to EDC (via interosseous membrane) FDS-IV to EPL and EIP (via interosseous membrane)

A 65-year-old woman undergoes open reduction and internal fixation for the injury displayed in the x-ray study shown. Six weeks postoperatively, she has diffuse swelling, stiffness, and pain of the right upper extremity. Her skin appears shiny, and she has changes in the patterns of both hair and perspiration in the right upper extremity when compared with the left upper extremity. After injury, treatment with which of the following would most likely have decreased her risk for this complication? A) Amitriptyline B) Ascorbic acid C) Gabapentin D) Prednisone E) Pregabalin

B Complex regional pain syndrome (CRPS) is chronic pain that persists in the absence of ongoing cellular damage and is characterized by autonomic dysfunction, trophic changes, and impaired function. In the perioperative period, the physiologic consequences of CRPS in the upper extremity contribute to or create one or more of the following: clinically significant osteopenia, delayed bony healing or nonunion, joint stiffness, tendon adhesions, arthrofibrosis, pseudo-Dupuytren palmar fibrosis, swelling, and atrophy. The reported incidence of CRPS is 5.5 to 26.2 per 100,000 person-years, and the prevalence is reported as 20.7 per 100,000 person-years. Women are more frequently affected than men, with a ratio of 3:1 to 4:1; the upper extremity is involved more frequently than the lower extremity; and fracture is the most common causative event. Incidence of CRPS after distal radius fracture has been reported to be between 22 and 39%. In a double-blind, prospective, multicenter trial by Zollinger et al., 416 patients with 427 wrist fractures were studied for the effects of prophylactic vitamin C (ascorbic acid) on the risk of subsequent development of CRPS. Administration of 500 mg vitamin C daily was found to significantly decrease the incidence of CRPS in patients with distal radius fracture. The authors recommend treatment for 50 days. The other four medications listed have all been reported for the treatment of patients diagnosed with CRPS. None have been reported to decrease the incidence of CRPS when used prophylactically.

A 60-year-old woman is evaluated in the emergency department after she fell on her outstretched hand while playing tennis. Examination shows tenderness in the dorsal and volar aspects of the wrist. Which of the following bones was most likely fractured in this patient? A) Capitate neck B) Distal radius C) Lunate body D) Scaphoid wrist E) Triquetral ridge

B Distal radius fractures usually occur in adults older than 40 years and are more common in women than in men due to the higher incidence of osteoporosis in women. The most common mechanism is a fall on an outstretched hand. After distal radius fracture, the next most common fracture of the wrist is scaphoid, followed by triquetrum, trapezium, and lunate.

A 25-year-old man is scheduled to undergo muscle transfer with the gracilis muscle to restore finger flexion. To optimize function, the muscle should be inset under which of the following? A) Less tension than it was in the leg B) The same tension as it was in the leg C) More tension than it was in the leg D) No tension

B Functional muscle transfers are a way to restore motion that has been lost. The gracilis muscle is a common option for this kind of transfer. To optimize the outcome, the muscle should be inset at the same tension it was under in the leg. The physiologic basis for this technique is that muscle fibers function best at a particular length/tension relationship. Muscles are typically under ideal tension in their donor position. When transferred, a muscle can be placed under too much or too little tension. If a muscle is overstretched, there is little overlap of the actin and myosin units, and the contractile force is weak. If the muscle is under too little tension, the actin and myosin units aren't able to achieve maximal contraction. Insetting a muscle under no tension produces the same result as insetting it under less tension.

A 23-year-old man is brought to the emergency department after twisting the long finger of his dominant right hand while playing basketball. Physical examination shows dorsal instability of the proximal interphalangeal (PIP) joint. X-ray study shows a volar buttress fracture involving 40% of the articular surface of the base of the middle phalanx. X-ray joint reduction is attained by passively flexing the PIP joint to 30 degrees. Which of the following is the most appropriate management? A ) Dynamic force-coupler external fixation B ) Extension block splinting C ) Hemi-hamate reconstruction of the volar buttress D ) Open reduction and internal fixation of the fracture fragments E ) Volar plate arthroplasty

B If the PIP fracture/dislocation can be maintained stable with 30 degrees of flexion, then this is suitable to treat by extension block splinting. This will be the case with an approximate 40% volar articular fracture of the base of the middle phalanx. There is a risk of late flexion contracture if greater than 30 degrees of flexion is required to maintain PIP joint stability. More complex injuries may be treated with force-coupler dynamic splinting or with hemi-hamate reconstruction. For a larger fracture fragment with greater instability, open reduction and internal fixation may be required. Volar plate arthroplasty may be suitable with more chronic injuries and is limited to 60% of the articular surface.

A 24-year-old woman comes to the office because of severe thenar atrophy 3 years after failed repair of a low median nerve laceration. Tendon transfer to restore thumb opposition is planned. Optimal transfer would restore which of the following thumb functions? A) Palmar abduction, extension, supination B) Palmar abduction, flexion, pronation C) Palmar abduction, flexion, supination D) Radial abduction, extension, pronation E) Radial abduction, flexion, supination

B Injury to the median nerve, either by laceration or compression, results in thenar atrophy and loss of thumb opposition. Tendon transfer is the only reliable technique to restore thumb function in the face of severe, long-standing atrophy. Thumb opposition is a composite movement comprised of palmar abduction, flexion, and pronation. Opposition positions the thumb for grasp, but is not synonymous with it. There are numerous tendon transfer procedures described to improve thumb opposition (e.g., palmaris longus, abductor digiti minimi, flexor digitorum superficialis), but the most effective improve each of the three components. Thumb extension, supination, and radial abduction (in the plane of the hand) are not movements involved in opposition.

A 25-year-old right-hand dominant man is brought to the emergency department after sustaining a stab wound to the right arm in a bar fight. Physical examination shows a 2 × 1-cm laceration over the antecubital fossa. He is unable to flex the interphalangeal joint of the thumb and the proximal interphalangeal joint of the index finger. Which of the following nerves is most likely injured? A) Lateral antebrachial cutaneous B) Median C) Musculocutaneous D) Radial E) Ulnar

B Median nerve palsy is marked by the inability to oppose the thumb or flex the thumb at the interphalangeal joint. The inability to flex the index finger at the proximal interphalangeal joint is also noted. The lateral antebrachial cutaneous nerve provides sensory innervation to the lateral aspect of the arm. The median antebrachial cutaneous nerve innervates the skin of the anterior and middle surfaces of the forearm to the level of the wrist. This nerve does not innervate any muscles. Radial nerve palsy is marked by the inability to extend the fingers, thumb, and wrist. Patients with radial nerve palsies have difficulty grasping objects. The results of tendon transfers to restore function in patients with radial nerve palsies are among the best and most predictable outcomes. Ulnar nerve palsy symptoms include a ?claw? deformity, with flexion deformities of the ring and little fingers. In later stages, profound muscle wasting of the both hypothenar eminence and the first web space is seen.

A 25-year-old man who is a graduate student comes to the office for evaluation of the right ring finger 4 weeks after sustaining an axial impact. A diagnosis of soft-tissue mallet finger is noted. Which of the following is the most appropriate treatment in this patient? A) Arthrodesis B) Orthosis C) Pinning in extension D) Tendon grafting E) Terminal tendon repair

B Most of these injuries even after a month will respond to splinting of the DIP joint in extension for 6 weeks. Any residual extension lag is largely an aesthetic concern and rarely will require further treatment. The operative treatment of soft-tissue mallet injury may lead to unacceptable complications while splinting may lead to skin irritation but little else. The preferred treatment at 4 weeks is closed reduction and splinting. Surgical treatment may be considered for more chronic injury greater than 3 months. Pinning in extension may be appropriate for a small subgroup of patients with work-related inability to wear an orthosis (such as surgeons).

Which of the following is the earliest one might expect to find electromyographic changes after suspected median nerve damage during carpal tunnel release? A) 1 Week B) 3 Weeks C) 5 Weeks D) 7 Weeks

B Patients with nerve injuries can be evaluated by nerve conduction velocities and electromyography (EMG). Abnormal conduction velocities are associated with decreased amplitude, decreased velocity, and increased latency. Sensory nerve latency above 3.5 ms and/or motor nerve latency above 4.5 ms are considered abnormal. Muscle changes assessed by EMG are typically altered later (2 to 3 weeks after injury) in the clinical course and consist of the presence of fibrillation potentials and decreased motor unit potential recruitment. Though the sensitivity of these studies may be somewhat low, at around 66% when using conduction velocity and latency, the specificity has been reproducibly near 95%.

A 25-year-old man comes to the office after sustaining a deep laceration to the elbow. Physical examination shows decreased function of the ulnar nerve, and the patient is taken for operative exploration and repair. Following proximal and distal dissection, a 1-cm gap between the proximal and distal nerve ends persists. Which of the following is the most appropriate next step in management? A) Nerve transfer B) Nerve transposition C) Polyglycolic acid nerve conduit D) Primary repair E) Sural nerve grafting

B Principles of microsurgical nerve repair include the use of meticulous and atraumatic technique with adequate magnification, microsurgical instruments, and sutures. A primary repair is performed whenever possible, provided that the repair is tension-free in order to maximize perfusion to the repair site. In this patient, a 1-cm nerve gap in the ulnar nerve was present even after mobilizing the proximal and distal nerve ends. In this situation, the ulnar nerve may be transposed anteriorly, which would shorten the distance between the nerve ends and allow for primary repair. Nerve transfers are indicated in very proximal nerve injuries where a proximal stump is unavailable for primary repair or grafting, or when a very long nerve gap is present where there would be a concern that target muscle denervation might occur prior to nerve regeneration. Polyglycolic acid nerve conduits are bioabsorbable tubes through which nerve regeneration occurs. They represent an option for nerve reconstruction without any associated donor site morbidity when a nerve gap is present in order to achieve a tension-free repair. Although primary nerve repair is preferable to the use of a graft/conduit, doing so in the setting of this patient's 1-cm nerve gap would not result in a tension-free repair. Autologous nerve grafting, such as with the sural nerve, is an option for nerve reconstruction when a nerve gap is present in order to achieve a tension-free repair.

A 21-year-old man comes to the office after injuring the right long finger while playing football. On physical examination, the patient is unable to flex the distal phalanx. Hollowness is noted along the volar aspect of the finger, with pain in the palm upon palpation. X-ray studies show no abnormalities. Which of the following is the latest at which primary repair is expected? A) 1 day B) 1 week C) 1 month D) 2 months E) 3 months

B Rupture of the flexor digitorum profundus (FDP) tendon from its distal attachment is commonly known as jersey finger. The injury is often overlooked by players and trainers and misdiagnosed as a sprained finger, but it requires more urgent management than these minor injuries. Jersey finger occurs when a flexed distal interphalangeal (DIP) joint is suddenly and forcefully hyperextended, leading to rupture of the FDP tendon at its insertion on the distal phalanx. FDP injuries can be classified based upon the degree of tendon retraction, as described in Leddy and Packer's grading scheme: Type I injuries involve retraction of the profundus tendon all the way to the palm, with associated injuries to the vincula longus and vinculum brevis. Injuries to the vincula disrupt the blood supply to the tendon, necessitating surgical repair within 7 days to avoid necrosis of the tendon and a permanent contracture deformity. Type II injuries involve retraction of the tendon to the proximal interphalangeal (PIP) joint. The tendon stump is held in place by the vincula longus, which are often intact. An avulsion fracture sometimes occurs with type II injuries, and often becomes trapped in the A2 pulley. Without an observable bony fragment on x-ray study, it is impossible to determine the degree of retraction; thus, all type II injuries should be surgically repaired within 7 days. Type III injuries involve a large avulsion fragment that is often intraarticular. The bony fragment prevents retraction past the A4 pulley and holds the tendon in near-anatomic position, obviating the need for urgent repair. Type III injuries are amenable to repair within 2 to 3 months. Type IV injuries are type III lesions with the addition of an avulsion of the FDP tendon from the fracture fragment. Type IV injuries are rare but require urgent repair because of the disruption to the tendon's blood supply.

A 50-year-old woman previously diagnosed with left forearm compression neuropathy of the superficial radial nerve comes to the office for examination. The patient has not responded to 7 months of conservative management which consisted of NSAID therapy, steroid injection, a trial of splinting, and activity modification. Operative treatment is planned. Fascial release between which of the following two tendons is most appropriate in this patient? A) Abductor pollicis longus and extensor pollicis brevis B) Brachioradialis and extensor carpi radialis longus C) Extensor carpi radialis longus and extensor carpi radialis brevis D) Flexor carpi radialis and abductor pollicis longus E) Flexor carpi radialis and brachioradialis

B Superficial radial nerve compression of the forearm occurs most frequently at the posterior border of the brachioradialis where the nerve transitions from a deeper, subfascial position to a more superficial, subcutaneous location. Also known as Wartenberg syndrome, patients may present with pain, numbness, or tingling over the dorsal radial hand radiating to the dorsal thumb and index finger. Symptoms of superficial radial nerve compression may be confused with symptoms of de Quervain's tenosynovitis. In addition, both conditions may coexist simultaneously. Patients diagnosed with superficial radial nerve compression are initially treated conservatively since this approach is successful in relieving symptoms in the majority of cases. Conservative management consists of rest, splinting, removal of external compression source (such as a tight wristwatch band, bracelet, or handcuffs), and nonsteroidal anti-inflammatory medications. Surgery is indicated when conservative measures fail. Surgical decompression involves release of the fascia between the brachioradialis and extensor carpi radialis longus tendons. It is at this interval that the nerve transitions from deep to superficial and prone to compression. The other responses do not reflect the correct surgical anatomy of this condition.

A 35-year-old man has clawing of all four fingers of the right hand 2 years after repair of a forearm laceration that injured the median and ulnar nerves. Photographs are shown. Both nerves were repaired shortly after the injury. All fingers have full passive range of motion. An extensor carpi radialis longus transfer is planned to correct the clawing of all four fingers. Which of the following donor sites is most likely to provide sufficient tendon graft for this procedure? A) Abductor digiti quinti B) Extensor digitorum longus C) Flexor digitorum superficialis of the long finger D) Palmaris brevis E) Pronator teres

B The Brand transfer uses the extensor carpi radialis longus or brevis as a donor motor to correct clawing of the fingers. It can be used to correct ulnar (ring and small finger) clawing or clawing of all four fingers. In either case, a tendon graft is needed to bridge the gap between the native distal limit of the extensor carpi radialis longus or brevis (on the index or long finger metacarpal base, respectively) to the transfer insertion on the lateral band at the proximal phalanx level. The transfer can be passed through the interosseous membrane in the forearm and then through the carpal tunnel or the intermetacarpal spaces in the hand. The transfer must pass volar to the deep transverse metacarpal ligament to have the correct vector of pull. The extensor digitorum longus provides four tendon slips distally, each with its own paratenon, but has one tendon coming off the muscle proximally. A photograph is shown. There is minimal donor site morbidity in the foot due to the retained function of the extensor digitorum brevis. Palmaris brevus has no tendon and cannot be used as a graft donor. Abductor digiti quinti can be used as a donor muscle for thumb opposition transfer, most commonly in children. It has a very short tendon and cannot be used for anti-claw transfers. In addition, due to this patient's injury, the abductor digiti quinti is likely denervated and would not be functional for a transfer. Flexor digitorum superficialis (FDS) of the long finger can be used for anti-claw transfers. The tendon can be split along the plane of Camper's chiasm to create two distal slips for insertion. It is well suited as an anti-claw transfer in patients with isolated ulnar nerve palsy. The FDS tendon cannot be split into four slips for insertion, as would be needed in this patient, and the power of one FDS muscle is insufficient to correct clawing in four fingers. In addition, the FDS resides superficial to the median nerve. In a patient who sustained a forearm laceration with injury to the median nerve, the overlying FDS is also likely to have been injured. Pronator teres is the most common donor motor to restore wrist extension in patients with radial nerve palsy. It has a very short tendon and is not used for anti-claw transfers.

A 45-year-old right-hand-dominant man who is a tennis player is evaluated because of a 6-month history of pain in the right lateral elbow. He has pain when lifting objects, and the pain radiates to the forearm. Physical examination shows tenderness just distal and anterior to the lateral epicondyle. Which of the following muscles is most likely affected? A) Brachioradialis B) Extensor carpi radialis brevis C) Extensor carpi radialis longus D) Extensor carpi ulnaris E) Extensor digiti minimi

B The extensor carpi radialis brevis (ECRB) origin is the primary muscle involved in lateral epicondylitis. The undersurface is avascular, making it a potential site for degeneration and partial tears. The ECRB shares a common origin with the extensor carpi ulnaris, extensor carpi radialis longus, and brachioradialis. The extensor digiti minimi also originates from the lateral epicondyle and has been involved in some cases of lateral epicondylitis, but not as commonly as the ECRB.

A 24-year-old man is evaluated because of a 1-cm metacarpal defect after sustaining a gunshot wound to the hand. In addition to operative fixation of the fracture, which of the following materials placed into the defect is most likely to promote osteogenesis? A) Calcium hydroxyapatite B) Cancellous autograft C) Cortical allograft D) Demineralized bone matrix E) Methylmethacrylate

B The material that will most likely provide osteogenesis is cancellous autograft. Bony defects can be constructed by a variety of methods, and there has been an increase in the number of biomaterials that can be used. Autograft bone is obtained from the same individual, while allograft bone is obtained from another human source (i.e., cadaveric or donor). Demineralized bone matrix does not contain calcium, but retains growth factors and proteins as the nonmineralized components of bone. Calcium hydroxyapatite is a bone substitute that mimics bone in mineral structure, and gradually becomes replaced with native bone. Methylmethacrylate is used in orthopedic bone cement, and is not biodegraded or replaced, but can provide rapid structural support. Osteoconduction refers to the replacement of the graft material through a process known as creeping substitution, where native cells from the surrounding bone break down the material and replace it with new bone. This is the primary mechanism of healing of cortical bone grafts. It is also seen in biocompatible materials that are replaced with bone, such as calcium hydroxyapatite. Osteoinduction refers to the stimulation of bone-forming cells from surrounding host tissues, resulting in activation of progenitor cells and differentiation into osteoblasts, leading to the creation of new bone. This process occurs due to growth factors that are present in the graft material, and can be seen in cancellous bone grafts, as well as demineralized bone matrix, which contains growth factors. Osteoinduction also occurs with cortical grafts, although to a lesser extent. Osteogenesis refers to new bone formation, which is provided from surviving cells within the graft material. In order for osteogenesis to occur, viable cells must be transferred with the graft. This is seen in autograft materials, but not in allograft materials, which are processed and may be decellularized. Cells contained within autografts can survive and produce new bone. Vascularized bone transfer may give rise to more cell viability than traditional autografts, because it maintains perfusion to the grafted bone, rather than relying on nutrients from the bed.

A 24-year-old man comes to the office because of numbness and difficulty moving his ring and little fingers 5 months after cutting his upper arm on broken glass. Current physical examination shows inability to abduct and adduct the ring and little fingers. Sensation to light touch is diminished. Following exploration and resection of a painful, traumatic neuroma, there is a 5-cm gap in the ulnar nerve proximal to the elbow. Which of the following is the most appropriate management to restore intrinsic muscle function? A) Cadaveric nerve allografting B) Nerve transfer C) Sural nerve grafting D) Use of nerve conduit E) Vascularized nerve grafting

B The most appropriate management for restoration of intrinsic muscle function is nerve transfer. In nerve injuries resulting in complete transection of the nerve, wallerian degeneration occurs at the site of transection, and Schwann cells in the distal nerve segment undergo apoptosis. With prolonged denervation, decreased regenerative ability with limitation in motor recovery is noted. Optimal functional recovery is dependent upon adequate reinnervation of the motor end plates and target muscles by regenerating motor axons. Over time, loss of target motor end plates via degeneration and fibrosis and replacement of muscle fibers by fat cells occur. Nerve regeneration occurs at a rate of approximately 1 mm daily or 1 inch monthly. In a high injury to the ulnar nerve, the distance from the proximal motor axons to the intrinsic musculature precludes timely reinnervation, and intrinsic recovery is generally poor. Reinnervation of the muscle ideally should be completed within 12 to 18 months following injury to allow for recovery. In the patient who has had the delayed symptoms and high ulnar nerve injury described, the time to recovery of intrinsic function will be greater than 2 years if the injury is reconstructed directly. This estimate is based on the elapsed time and distance to the target muscles. Nerve transfer involves the use of a noncritical or expendable donor motor nerve to reinnervate a missing function. The selection of an available motor nerve donor that is closer to the target muscle can decrease the time needed for reinnervation of the muscle and help to ensure recovery before irreversible changes occur. In the scenario described, the distal portion of the anterior interosseous nerve can be used as a donor nerve to reinnervate the ulnar motor branch. Transfer of the distal anterior interosseous nerve to the motor branch of the ulnar nerve will provide motor neurons in a more distal location to reinnervate the intrinsic muscles in the desired time frame. Nerve grafting is the most appropriate management to bridge a nerve gap when direct repair is not possible. This would be indicated if the circumstances dictated that muscle reinnervation could occur in an adequate or timely fashion, such as in a more recent injury or in a nerve gap that is closer to the target muscles. Nerve grafting may be performed for sensory recovery, but motor reinnervation is unlikely to occur in the scenario described. The sural nerve provides a good source of nerve autograft that is long (up to 40 cm) and of reasonable diameter (2 to 3 mm), with minimal donor site morbidity. A variety of artificial nerve conduits have been developed to avoid the need to harvest nerve grafts. Nerve conduits of polyglycolic acid and collagen have been developed to bridge nerve gaps; however, recovery is not as effective as compared with autogenous nerve grafts. Typically, conduits are used for sensory nerves in noncritical areas. Gaps of up to 2 to 3 cm can be bridged. There has been recent interest in processed preserved nerve allografts which are obtained from cadaveric sources. Studies suggest that allograft nerves may regenerate motor neurons better than nerve conduits, but autologous nerve grafting remains the gold standard. Typically, nerve allografts are more useful for short segmental nerve gaps. A vascularized nerve graft allows transfer of the blood supply along with the nerve. This decreases the need for revascularization from the surrounding tissues and may be useful for grafting long nerve gaps with badly scarred or irradiated beds.

A 23-year-old man is brought to the emergency department after being stabbed with a knife in a fight. The patient appears alert and in no distress. Physical examination shows a 2-cm puncture wound just above the midclavicular line. Decreased strength with wrist flexion, weakness of index, long, ring, and little finger flexion at the metacarpophalangeal (MCP) and proximal interphalangeal (PIP) joints, and difficulty with palmar abduction against resistance are noted. Finger, thumb, and wrist extension is intact. Elbow flexion and extension are normal. Sharp injury to which of the following structures is most likely in this patient? A ) Lateral cord B ) Medial cord C ) Median nerve D ) Middle trunk E ) Ulnar nerve

B The patient described demonstrates median and ulnar nerve weakness after a sharp injury to the brachial plexus. The radial nerve function appears to be preserved. The medial cord gives fibers to the medial pectoral nerve, the medial brachial and antebrachial cutaneous nerves, the median nerve, and the ulnar nerve. Therefore, this is the appropriate response. The lateral cord is not appropriate because although that structure does give fibers to the median nerve, it does not branch to the ulnar nerve. The lateral cord terminates in the lateral pectoral nerve, musculocutaneous nerve, and the median nerve. Also of note, as the patient has intact elbow flexion, the musculocutaneous nerve is presumably intact in the scenario described. The median nerve in isolation is not appropriate as well for similar reasons. This patient has median and ulnar nerve deficits. The middle trunk contains fibers that go to the radial and median nerves, but not the ulnar. This is not appropriate, therefore, as well. The posterior cord gives off the radial nerve, axillary nerve, subscapular nerves, and the thoracodorsal. It does not contribute to the median or ulnar nerve. At the midclavicular line, the ulnar nerve has not yet formed.

A 22-year-old man comes to the emergency department after sustaining a laceration to the dorsal thumb by punching a glass door. Radial nerve block is planned during surgical repair. Which of the following is the approximate distance proximal to the radial styloid in which the superficial branch of the radial nerve pierces the deep fascia? A) 0 to 4 cm B) 5 to 9 cm C) 10 to 14 cm D) 15 to 19 cm

B The superficial branch of the radial nerve runs below the brachioradialis muscle in the mid-forearm, later becoming sub-fascial between the brachioradialis and extensor carpi radialis longus (ECRL) muscles. Approximately 8 to 9 cm proximal to the radial styloid, the superficial branch of the radial nerve (SBRN) becomes subcutaneous, piercing the fascia. The ideal location of infiltration for a radial nerve block is at the sub-fascial location just before the nerve becomes subcutaneous.

A 51-year-old woman is evaluated because of numbness and tingling of the dorsal and palmar aspects of the left hand, extending to the ring and little fingers, with worsening symptoms at night. Physical examination shows weakness of finger abduction in the hand. Which of the following is the most likely electrodiagnostic finding? A) Decreased median conduction velocity from above elbow to wrist B) Decreased ulnar conduction velocity from above elbow to wrist C) Decreased ulnar conduction velocity from below elbow to wrist D) Prolonged median sensory latency from wrist to digit E) Prolonged ulnar sensory latency from wrist to digit

B This patient exhibits symptoms of ulnar nerve compression at the cubital tunnel. Cubital tunnel syndrome is characterized by numbness and tingling in the ulnar nerve distribution (ulnar side of hand, involving little finger and ulnar half of ring finger) and can lead to intrinsic weakness. Compression occurs at the level of the elbow, with slowing of nerve conduction across the area of compression. Electrodiagnostic findings in ulnar nerve compression consist of decreased ulnar conduction velocity in the segment from above elbow to the wrist. Comparison of conduction velocities between above elbow to wrist and below elbow to wrist may show a difference, with nerve conduction being faster when measured from below the elbow, as the area of compression is not traversed. Conduction velocities from below elbow to wrist should not be affected. Ulnar nerve compression at the cubital tunnel can be distinguished from compression at the Guyon canal, as symptoms in the dorsal hand are not involved during nerve compression at the Guyon canal (at the level of the wrist), because the dorsal sensory branch of the ulnar nerve branches proximal to the wrist. Prolonged median sensory latency from wrist to digit is seen in carpal tunnel syndrome.

A 44-year-old woman is evaluated because of a 6-month history of pain in her right upper chest and back, intermittent coolness in her right hand, and numbness and tingling of her right ring and little fingers. Results of the Adson test show a decreased radial pulse on the affected side, and the Roos test reproduces the patient's symptoms on the affected side. Which of the following is the most appropriate next step? A) Decompression of ulnar nerve at elbow B) Noninvasive vascular study and electrodiagnostics C) Resection of anterior and middle scalene muscles D) Transaxillary resection of first rib E) Observation

B Three kinds of surgical procedures are employed to treat thoracic outlet compression syndrome (TOCS): transaxillary resection of the first rib, transcervical anterior and medial scalenectomies, and combined transaxillary first rib resection with immediate anterior and medial scalenectomies. This is the most complete procedure for total decompression of the thoracic outlet region. Because 70% of cases have soft-tissue involvement as the etiology of TOCS, current treatment includes transcervical anterior and middle scalenectomy in most TOCS cases. Prior to any surgery, patients are treated conservatively with an exercise program for TOCS involving scalene stretching, first rib intercostal relaxation, nerve gliding, muscle relaxants, and pain patches for painful myofascial trigger points. Unfortunately, these conservative treatment modalities may yield only limited temporary help. TOCS is usually classified in two groups. A neurogenic group comprises nearly 90% of all cases. This group usually has upper extremity pain, numbness, and tingling. A true vascular group comprises 10% of cases. Approximately 50% of patients still complain of coldness in the extremity. Approximately 40 to 50% of TOCS cases have concomitant peripheral nerve compression symptoms. Simple distal decompression of nerves will not usually lead to near-complete resolution of symptoms in cases of true TOCS. There are two tissue groups that cause TOCS: soft tissue and osseous structures. The soft-tissue group includes anterior and middle scalene and their sheath, ligaments, and bands. This group comprises at least 70% of all TOCS cases because of congenital and acquired changes in the soft tissues. The osseous group comprises 30% or less of all TOCS cases and includes cervical rib, changes in the first rib, and clavicle due to injury. Because TOCS can present with several different findings (including vascular and neurological compromise), it is advisable to work up these findings prior to committing to a treatment course. Noninvasive vascular studies and electrodiagnostics is the most reasonable first step in working up and treating these patients.

An 8-year-old girl is brought to the office for evaluation because her right forearm is severely contracted at the wrist and fingers. Upon questioning, the patient's parents note that she was treated 1 year ago for a humerus fracture. Physical examination shows a pronated forearm, flexed wrist, hyperextended metacarpophalangeal (MCP) joints, and flexed proximal (PIP) and distal interphalangeal (DIP) joints. She can make a full active fist. Which of the following is the most appropriate management? A ) Exploration at the level of the humerus fracture with neurolysis and/or repair of the radial nerve B ) Exploration of the forearm, neurolysis of the median and ulnar nerves, and muscle slide technique performed C ) Exploration of the upper arm with median and ulnar nerve neurolysis D ) Flexor carpi radialis to extensor carpi radialis brevis tendon transfer

B Volkmann described the sequelae of compartment syndrome following a supracondylar fracture of the humerus in a child with the development of a severely contracted and functionless forearm. The forearm is typically fixed in pronation, the wrist is flexed, and the hand is postured in the ?claw? position with the MCP joints hyperextended and the PIP and DIP joints flexed. The hand is usually insensate. Treatment depends on the severity of the deformity. Moderate contractures are treated with exploration and release of both the median and ulnar nerves and a tendon lengthening procedure (muscle slide). This condition is not due to a radial nerve injury; therefore, exploration of the fracture site and neurolysis of the radial nerve are not indicated. Tendon transfer is not appropriate because it does not address the nerve compression. Revascularization with a bypass graft is not needed because it is no longer ischemic.

A 45-year-old man has a mass in the left volar forearm. An MRI is shown. Oncologic resection will involve removing all muscles of the anterior forearm compartment. Which of the following techniques is most appropriate to reconstruct finger flexion postoperatively? A) Flexor digitorum profundus to superficialis transfer B) Flexor pronator slide C) Free gracilis innervated by a median nerve branch D) Oberlin-Mackinnon nerve transfer E) Pedicled latissimus dorsi muscle flap transfer

C A free gracilis muscle would provide a good strength and excursion match to the native finger flexors it would replace. Vascular and nerve connections could be performed in the distal upper arm, outside of the zone of resection. While individual finger flexion would not be restored, the patient would be able to make a composite fist after this surgery. The flexor digitorum profundus to superficialis transfer is used to treat flexor spastic contracture, typically with palmar hygiene issues, in a patient who still has some voluntary motor control. It requires the presence of flexor digitorum profundus and superficialis muscles, both of which would be removed as part of the anterior forearm compartment muscles. The Oberlin-Mackinnon nerve transfer transfers branches of the median nerve and ulnar nerve to the brachialis and biceps muscles, respectively. It is used to restore elbow flexion and would not provide finger flexion. A flexor pronator slide detaches the muscles originating from the medial epicondyle of the humerus and advances them distally. It is used to treat contractures of the fingers, often in patients with mild to moderate Volkmann contracture. In this patient, the flexor muscles have been resected, so this procedure is not possible. A latissimus dorsi muscle provides broad soft-tissue coverage for wounds and can also provide a strong flexion force. It cannot reach beyond the distal elbow. It can restore elbow flexion but cannot be used as a pedicled transfer to restore finger flexion.

A 30-year-old woman comes to the emergency department after cutting herself with a kitchen knife. Surgical exploration shows that the median nerve had been cut at the distal forearm. The distal nerve is relatively fixed, but the proximal nerve has been lifted and twisted. Immediate repair is planned. The motor fibers of the proximal nerve end are most likely in which of the following locations relative to the sensory fibers? A ) Dorsal and radial B ) Dorsal and ulnar C ) Volar and radial D ) Volar and ulnar

C At the level of the distal forearm, the median nerve is a mixed nerve comprised of both motor (20%) and sensory (80%) fibers. The motor fibers become the thenar branch, which innervates the abductor pollicis brevis (AbPB), opponens pollicis (OP), and flexor pollicis brevis (FPB). The FPB is located distal to the AbPB and OP and also has innervation from the ulnar nerve, which is why patients with median nerve injuries can sometimes still bring the thumb to the little finger. The thenar nerve can be injured with a carpal tunnel release procedure. After coursing into the carpal tunnel, these median nerve motor fibers leave the median nerve volar and radial to the sensory fibers of the median nerve through a variety of branching patterns. When patients have clean-cut injuries to nerves, the nerves can be repaired primarily. In a contaminated, dirty wound caused by a crush and mutilating injury, it can be difficult to determine the nonviable nerve required for debridement before repair. In the scenario described, the nerve ends are labeled with a permanent suture for later identification and repair at a second stage. In general, sensory-only nerves can be repaired by epineurial approximation, and group fascicular repair can be considered for mixed nerves.

Which of the following is the type of axon fiber (neuron) that is primarily involved with the autonomic changes that occur with complex regional pain syndrome (CRPS) type 1? A) A delta sensory B) Alpha motor C) C sensory D) Gamma motor E) Ia sensory

C C sensory fibers are responsible for a deeper, more non-localizable pain. C fibers can react to various stimuli, including thermal, mechanical, or chemical. C fibers respond to physiologic changes in the body, such as hypoxia, hypoglycemia, hypo-osmolarity, the presence of muscle metabolic products, and light or sensitive touch. Paul Sudeck noticed that CRPS demonstrates classic inflammatory signs such as pain, swelling, erythema, hyperthermia, and impaired function. However, clinical chemistry markers of inflammation are not elevated. These findings imply a neurogenic inflammation. C fibers have an afferent function in the mediation of pain (and itch), but also an efferent neurosecretory function. They release neuropeptides such as substance P and calcitonin-gene-related peptide (CGRP). The presence of these neuropeptides might explain trophic and autonomic symptoms such as swelling, erythema, and hyperhidrosis. Elevated CGRP levels are also associated with autonomic disturbances, mainly with hyperhidrosis. Also, a role for CGRP in hair growth is suggested, and substance P seems to be involved in the regulation of osteoclastic activity. Alpha motoneurons innervate muscle fibers of skeletal muscle and are directly responsible for initiating their contraction. When the central nervous system sends out signals to alpha neurons to fire, signals are also sent to gamma motoneurons to do the same. This process maintains the tautness of muscle spindles and is called alpha gamma co-activation. Without gamma motoneurons, muscle spindles would be very loose as the muscle contracts. Unrestricted alpha activity would not allow for muscle spindles to detect a precise amount of stretch and would not allow for optimization of muscle function. Ia sensory fibers are a type of proprioceptor that is found inside the muscle itself. They lie parallel to the contractile fibers, and give them the ability to precisely monitor muscle length. A delta fiber is a type of sensory nerve fiber. A delta fibers carry cold, pressure and some pain signals. Because A delta fibers have a higher conduction velocity, and are responsible for quick, shallow pain to a specific area. They are activated by a stimulus of weaker intensity, and are not responsible for the autonomic changes seen with CRPS.

Stellate ganglion blocks help alleviate the symptoms of complex regional pain syndrome in which of the following ways? A) Helping repair nerve injuries B) Increasing parasympathetic tone C) Reducing sympathetic tone D) Relaxing the muscles of the upper extremity E) Releasing the body's natural endorphins

C Complex regional pain syndrome (CRPS) is a long-term pain condition that is believed to result from dysfunction in the central or peripheral nervous systems. CRPS is characterized by pain, swelling, or stiffness in the affected hand or extremity. The pain may be out of proportion to the injury that triggers it. There are two types of CRPS. In type I, there is no identifiable nerve injury. In type II, there is an identifiable nerve injury. The name of this disorder was changed from reflex sympathetic dystrophy to CRPS because not all patients have increased sympathetic tone. Stellate ganglion blocks may be used to anesthetize the stellate ganglion, which is a cluster of sympathetic nerves at the base of the neck, in an effort to decrease the overactivity of the sympathetic nerves seen in CRPS. The sympathetic nervous system has been implicated in the pathophysiology of CRPS, and consequently, sympathetic nervous system blockade is widely used to treat CRPS. Sympathetic nervous system dysfunction is presumed to be an essential component of the syndrome, and sympathetic blockade has been recommended as early as possible to interrupt and reverse the process. The treatment effectively cuts the vicious cycle of pain, immobilization, and decreased joint motion. Stellate ganglion blocks do not act to increase parasympathetic tone, relax muscles, release endorphins, or repair injured nerves.

A 60-year-old woman presents with weakness and inability to fully extend the right dominant thumb at the interphalangeal joint. History includes a Colles fracture of the right wrist 6 months ago. Management of the fracture included cast immobilization. On physical examination, the patient's thumb is at 30 degrees of flexion. Finger metacarpophalangeal joint active extension is normal. The patient cannot extend or lift the thumb with her hand flattened on a table. With the thumb adducted, she can extend it to neutral. All thumb joints are supple and have full range of passive motion. Which of the following is the most likely cause of this patient's inability to extend the thumb? A) Intersection syndrome B) Radial nerve palsy C) Rupture of the extensor pollicis longus D) Saddle deformity of the basal joint E) Trigger thumb with locking

C Extensor pollicis longus (EPL) rupture is most commonly caused by late effects of distal radius fractures. Devascularization is the most likely cause leading to attritional rupture. Rupture can present 2 weeks to 11 months after fracture; the average is 7 weeks. This patient can extend her thumb when it is adducted because of connections of the intrinsics with the dorsal apparatus. Tendon transfer of the extensor indicis proprius to distal EPL stump is the first-line treatment. Other causes of EPL rupture include synovitis from rheumatoid arthritis and lupus causing friction at Lister's tubercle, steroid injections, excessive abnormal wrist motion, bony spurs following distal radius and scaphoid fractures, scaphoid nonunion, misplaced external fixator pin, and subluxation of the distal ulna. A trigger thumb would more likely have pain and tenderness on physical examination along the volar flexor sheath. A locked thumb would not have passive extensibility or active extension with adduction. Saddle deformity is seen on physical examination in advanced osteoarthritis of the basal joint. Interphalangeal joint flexion of the thumb is caused by zigzag longitudinal collapse with hyperextension of the metacarpophalangeal joint. Intersection syndrome is characterized by pain and tenderness along the radial side of the forearm where the abductor pollicis longus and extensor pollicis brevis muscles intersect. Radial nerve palsy would not only affect the thumb interphalangeal joint.

A 22-year-old laborer underwent four-strand and epitendinous repair of a Zone II flexor digitorum profundus (FDP) and flexor digitorum superficialis (FDS) injury to the long finger of the dominant right hand 5 days ago. Early active motion therapy protocol is selected for rehabilitation, by which the injured finger is passively flexed and the wrist extended, with the patient then asked to actively maintain a flexed grasp. In contrast to the rubber band Kleinert technique, this protocol is most likely to have which of the following effects on the repaired finger? A) Decreased risk of tendon rupture B) Greater risk of finger flexion contracture C) Increased FDP and FDS excursion D) More tendon adhesions E) Prolonged tendon repair softening

C Low force and moderate excursion therapy protocols continue to be the most effective protocol following flexor tendon repairs. However, increasing the applied force to the repair site during postoperative rehabilitation beyond 5 N does not accelerate accrual of repair site strength after a multistranded repair. In vivo repair, results have shown that early active mobilization may limit tendon end softening and loss of repair strength that generally occurs after the first 7 days. The mode of rehabilitation described was popularized by Strickland and has been shown to improve outcomes of Zone II repairs, probably due to both increased absolute as well as relative tendon excursions. However, in order to reduce the potential increased repair rupture rate, at least four-strand repair is required. This increased excursion leads to fewer tendon adhesions. Because the fingers are flexed by rubber bands, the Kleinert technique predisposes the patient to flexion contractures. A combination of the Duran passive range of motion and the Kleinert technique improves the results.

Volkmann ischemic contracture occurs when forearm compartment pressure is above 30 mmHg for a minimum of approximately how many hours? A ) Less than 2 B ) 3-5 C ) 6-12 D ) 18-24 E ) Greater than 24

C Permanent neuromuscular damage occurs at 12 hours, leading to subsequent Volkmann contracture. Timeline of ischemia, shown experimentally, is capillary endothelial damage at 3 hours; partially reversible muscle and nerve injury occurs at 6 hours.

A 25-year-old woman comes to the office because of nerve compression of the right upper extremity. Electromyography and nerve conduction studies are planned. Which of the following is the most likely indicator of motor axon loss in this patient? A) Absent polyphasic waveforms B) Decreased distal motor latency C) Fibrillation potentials D) Increased amplitude E) Increased conduction velocity

C Specific electrodiagnostic criteria indicate axonal loss: nerve conduction study amplitudes are decreased, conduction velocity is slowed, distal latency is prolonged, and fibrillation potentials and polyphasic waveforms are present.

A 62-year-old woman is evaluated for acute rupture of an extensor tendon after undergoing closed treatment of a nondisplaced distal radius fracture 6 months ago. The tendon most likely to be involved is located in which of the following extensor compartments? A) First B) Second C) Third D) Fourth E) Fifth

C Spontaneous rupture of the extensor pollicis longus (EPL) tendon is reported to occur in approximately 0.3 to 5% of nondisplaced or minimally displaced distal radius fractures, but it can also occur without trauma or in patients with inflammatory conditions such as rheumatoid arthritis. This is thought to arise from a loss of vascularity and atrophic changes in the compartment, and, because the tendon substance is usually degenerated, primary repair of the tendon is usually not possible. Tendon transfer using the extensor indicis proprius is the standard of care. Spontaneous rupture of other extensor tendons can occur in association with other conditions (e.g., rheumatoid arthritis), but would be exceedingly uncommon in the clinical scenario presented. The EPL passes through the third extensor compartment. Extensor tendon-compartment relationships include the following: First - abductor pollicis longus, extensor pollicis brevis Second - extensor carpi radialis longus, extensor carpi radialis brevis Third - extensor pollicis longus Fourth - extensor digitorum communis, extensor indicis proprius Fifth - extensor digiti minimi Sixth - extensor carpi ulnaris

A 37-year-old woman presents with sharp lateral elbow pain sustained when lifting a garbage bag out of a can at work. Which of the following muscles is the most likely cause of this patient's pain? A) Anconeus B) Brachioradialis C) Extensor carpi radialis brevis D) Extensor carpi radialis longus E) Extensor carpi ulnaris

C The common extensor tendon attaches the extensor carpi radialis brevis (ECRB), extensor digitorum communis (EDC), extensor digiti minimi, and extensor carpi ulnaris to the lateral epicondyle. Among these forearm extensor muscles, pathology found in the attachment of ECRB and EDC at the lateral epicondyle (LE) is commonly cited as a reason for pain at the LE. While the extensor carpi radialis longus (ECRL) is a wrist extensor and may be a source of lateral elbow pain with strain, it does not originate from the lateral epicondyle. The ECRL muscle was observed to originate from the distal aspect of the supracondylar ridge mainly as a muscular attachment. The brachioradialis is an elbow flexor and originates off of the lateral column of the distal humerus. It is not involved with lateral epicondylitis. The anconeus is a small muscle which originates off of the lateral epicondylitis and its main function is to assist with elbow extension.

A 35-year-old, right-hand-dominant man comes to the office because of passively correctable clawing of all four fingers of the right hand 1 year after he sustained a stab wound to the proximal right forearm that lacerated the ulnar nerve and artery, median nerve, flexor digitorum superficialis (FDS), flexor digitorum profundus, flexor carpi radialis (FCR), and flexor carpi ulnaris (FCU). Each of the injured structures was repaired primarily on the day of injury. A photograph is shown. Which of the following tendons is the most appropriate donor to address the clawing deformity? A) Abductor pollicis brevis B) Brachioradialis C) Extensor carpi radialis brevis D) FCRbr> E) FDS-3 to the long finger

C The flexor digitorum superficialis and flexor carpi radialis musculotendinous units were lacerated in the original injury. They would not be appropriate donor motors due to this. Brachioradialis transfer to the flexor pollicis longus transfer has been reported for patients with cervical spine injuries, but it is not used for transfers to restore intrinsic muscle function. The abductor pollicis brevis cannot be used to correct a claw deformity due to its small size and position in the thenar eminence; in addition, for this patient, its innervation was injured in the original trauma. Both the extensor carpi radialis longus and brevis have been described as a tendon transfer. Neither muscle has been affected by the initial injury. Whichever tendon is not harvested can power wrist extension along with the extensor carpi ulnaris. The tendon does need to be elongated with a graft.

A 22-year-old man comes to the office because of injury to the right index finger flexor tendons in Zone II. During open repair, which of the following flexor tendon pulleys arise from volar plates? A) A1, A2, and A3 B) A1, A3, and A4 C) A1, A3, and A5 D) A2 and A4 E) A4 and A5

C The flexor tendons are bound within a fibro-osseous sheath, with pulleys essential to prevent bowstringing (and consequent poor force transfer). The pulley system includes both annual and cruciate pulleys. Pulleys A2 and A4 attach to bone. Pulleys A1, A3, and A5 are attached to the volar plates at their respective joints. The strength of pulleys, in order from strongest to weakest, are the A2, A1, and A4 pulleys. The pulleys attached to bone have a higher breaking strength compared with those attached to the volar plates.

The metacarpophalangeal (MCP) joint of the thumb is which of the following types of joint? A) Ball-and-socket B) Condyloid C) Hinge D) Pivot E) Saddle

C The metacarpophalangeal (MCP) joint of the thumb and interphalangeal joints of the index through little fingers are hinged joints and allow flexion and extension only. Lateral forces can disrupt the collateral ligaments, resulting in partial or full tears. Condyloid joints allow flexion and extension, abduction and adduction, and circumduction, and they can be seen in the MCP joints of the index through little fingers and in wrist joints. Saddle joints allow flexion and extension, abduction and adduction, and circumduction, and they can be seen in the carpometacarpal joint on the thumb. Ball-and-socket joints allow flexion and extension, abduction and adduction, and internal and external rotation, and they can be seen in the shoulder and hip joints. Pivot joints allow rotation and are seen in the atlas and axis bones.

A 50-year-old man who is homeless is brought by ambulance to the emergency department. His blood alcohol concentration is 325 mg/dL. Examination of the right hand and forearm shows absent palpable pulses at the radial and ulnar arteries. Compartment pressure is 55 mmHg. Which of the following nerves is most likely irreversibly affected in this patient? A) Lateral antebrachial B) Medial antebrachial C) Median D) Radial E) Axillary

C The most appropriate answer is median. Pathophysiology of Volkmann's contracture begins with the deep and central muscles, which include flexor digitorum profundus and flexor pollicis longus. The next affected is the middle layer, which includes flexor digitorum superficialis and pronator teres and then the wrist flexors. Lastly, the extensor forearm is affected. In terms of nerve sensitivities, beginning at 30 mmHg, there are decreased conduction velocities. At 50 mmHg, there is no conduction. After 8 hours, there is irreversible damage. The median nerve is affected before the ulnar nerve. The radial nerve is dorsal and not in the deep compartment. Both antebrachial nerves are superficial. The axillary nerve does not go to the forearm. Alcohol is a clear comorbidity in this patient and therefore the timing is unknown. The pulselessness in this case indicates a late finding of compartment syndrome. Pain out of proportion along with paraesthesias and pressure is an early sign. Other late signs include pallor and paralysis.

A 49-year-old man comes to the office because he has been unable to extend the wrist, fingers, and thumb of his right hand since fracturing his humerus 16 months ago. He underwent open reduction and internal fixation at that time. The fracture healed well. There has been no change in function since the procedure. Physical examination shows the patient is unable to actively extend the wrist, fingers, and thumb. Tendon transfers are planned. Which of the following is the most appropriate muscle to transfer for restoration of finger extension? A) Brachioradialis B) Extensor carpi radialis longus C) Flexor carpi ulnaris D) Palmaris longus E) Pronator teres

C The most appropriate muscle to transfer for restoration of finger extension is the flexor carpi ulnaris. The radial nerve can be injured as a result of humerus fracture and/or surgery as it crosses the spiral groove of the humerus. The resultant radial nerve palsy will cause inability to extend the wrist, fingers, and thumb. Reinnervation of the muscle ideally should be completed within 12 to 18 months following injury to allow for recovery. In this patient who has radial nerve palsy after humerus fracture, the time following injury has been too long, so nerve repairs or nerve transfers are not a viable option, and tendon transfer is the procedure of choice. Tendon transfer involves the use of a noncritical or expendable donor tendon to provide a missing function. The tendon to be transferred should have adequate strength and range of motion to provide the desired function. Ideally, the tendon used should have synergistic action and allow for tenodesis to facilitate reeducation. Transfer of the flexor carpi ulnaris to the extensor digitorum communis will provide extensor function of the fingers, as it has adequate power and excursion and takes advantage of the linkage between wrist flexion and finger extension. Other typical tendon transfers for finger extension in radial nerve palsy include the flexor carpi radialis and the flexor digitorum superficialis. The brachioradialis is a radial nerve innervated muscle and will not be functioning in this patient who has a high radial nerve palsy. In low radial nerve palsies, it can be used to restore thumb extension. The brachioradialis can also be used to restore finger or wrist extension, as well as finger or thumb flexion in the appropriate patient. The extensor carpi radialis longus is not functional in this patient with radial nerve palsy. It can be used as a transfer for finger flexion in the appropriate patient. The palmaris longus does not have sufficient power to provide finger extension. It can be used as a transfer for thumb extension. The pronator teres has adequate power but less excursion. It is typically used to restore wrist extension rather than finger extension.

A 13-year-old boy is brought to the office because he has difficulty opening his hand and extending his fingers. History includes release of the forearm compartments to treat a pulseless hand following a supracondylar humerus fracture 2 years ago. On physical examination, passive extension of the fingers is restricted when the wrist is fully extended; it improves with full wrist flexion. Which of the following muscles is the most likely cause of the limitation described? A) Flexor carpi radialis B) Flexor carpi ulnaris C) Flexor digitorum profundus D) Flexor digitorum superficialis E) Lumbricals

C The most likely cause of the restricted finger extension described is fibrosis of the flexor digitorum profundus muscle. The patient exhibits Volkmann ischemic contracture as a complication of late treatment (over 24 hours from the time of initial ischemia) of arterial compromise associated with the fracture. The muscle groups at the greatest risk during these ischemic episodes are within the deep flexor compartment of the forearm. This risk occurs because the arterial supply is relatively distant from the usual site of occlusion and because this compartment is relatively less distensible. In the scenario described, the flexor digitorum profundus and flexor pollicis longus are at the greatest risk. Superficial muscle groups such as the flexor carpi radialis, flexor carpi ulnaris, and the flexor digitorum superficialis typically recover some function and do not lead to contractures in the forearm. Likewise, the small muscles of the hand, such as the lumbricals, tend to be less severely injured than the deep compartment of the forearm.

A 30-year-old woman comes to the office because of burning pain in the left wrist 5 months after discharge from the hospital. During her visit, a catheter was inserted in the left radial artery for arterial line monitoring in the intensive care unit. On examination, point tenderness and Tinel sign are noted over the volar radial aspect of the forearm, just ulnar to the radial artery, and overlying the flexor carpi radialis tendon. Which of the following is the most likely nerve of origin for the suspected condition? A) Anterior interosseous B) Median C) Musculocutaneous D) Radial E) Ulnar

C The neuroma is of the lateral antebrachial cutaneous nerve, which innervates the area in question, over the volar forearm, including the skin overlying the flexor carpi radialis (FCR) tendon. The lateral antebrachial cutaneous nerve is the continuation of the musculocutaneous nerve in the forearm. The dorsal radial sensory nerve innervates the skin overlying the dorsal and radial aspects of the wrist and does not innervate the skin overlying the FCR tendon. Therefore, the radial nerve is not appropriate. The ulnar and median nerves are not appropriate, as they give off sensory contributions in the palm and fingers. The anterior interosseous nerve is mainly a motor nerve in the forearm, finally sending off branches to the volar capsule of the wrist.

A 40-year-old man who is an avid cyclist comes for evaluation of a 5-week history of numbness and tingling of the ring and little fingers of the left hand. Conservative management has failed. Physical examination shows weakness of grip strength. Sensation over the dorsal ulnar aspect of the hand is intact, and results of elbow flexion testing are negative. Which of the following structures are most appropriate for surgical release? A ) Osborne ligament and arcade of Struthers B ) Osborne ligament and volar carpal ligament C ) Pisohamate ligament and volar carpal ligament D ) Transverse carpal ligament and arcade of Struthers E ) Transverse carpal ligament and pisohamate ligament

C The patient described has ulnar tunnel syndrome or compression of the ulnar nerve in the Guyon canal. Sensory sparing on the dorsal ulnar aspect of the hand suggests a lesion distal to the origin of the dorsal cutaneous branch of the ulnar nerve. Nerve conduction studies and electromyography can be used to confirm the diagnosis. Conservative treatment includes activity modification, splinting, and administration of a nonsteroidal anti-inflammatory drug. The most appropriate management of this condition is exploration of the entire ulnar tunnel and release of the pisohamate and volar carpal ligaments. The ulnar tunnel, which is 4 to 4.5 cm in length, begins at the proximal volar carpal ligament and ends at the fibrous edge of the hypothenar muscles. Zone I is the region of the tunnel proximal to the bifurcation of the nerve. Zone II is the area around the deep motor branch, which ends at the pisohamate ligament, and Zone III is the area surrounding the superficial branch. The ulnar nerve courses between the volar carpal ligament and the transverse carpal ligament. The Osborne ligament and the arcade of Struthers are potential sites of compression of the ulnar nerve at the elbow, or cubital tunnel syndrome. The fascia of Osborne, or the cubital tunnel retinaculum, is a band bridging the two origins of the flexor carpi ulnaris muscle and the medial epicondyle. The arcade of Struthers is 8 to 10 cm proximal to the medial epicondyle and extends from the medial intermuscular septum to the medial head of the triceps. Release of the transverse carpal ligament is appropriate for management of compression of the median nerve at the wrist, or carpal tunnel syndrome.

A 17-year-old boy is brought to the emergency department 5 hours after he sustained a stab wound to the left dorsal forearm. On physical examination, he is unable to extend the thumb and metacarpophalangeal joints. Exploration of the wound for repair of a presumed nerve injury is planned. Proper exposure of the nerve is between which of the following muscle groups? A) Brachialis and triceps B) Brachioradialis and extensor carpi radialis longus (ECRL) C) Extensor carpi radialis brevis (ECRB) and ECRL D) Extensor digitorum communis and ECRB E) Flexor carpi radialis and pronator teres

D Inability to extend the thumb and metacarpophalangeal joints generally suggests an injury to the radial nerve. Wrist extension can be preserved because of the preservation of the extensor carpi radialis longus muscle innervation. In the forearm, the radial nerve can be best approached directly between the extensor digitorum communis and the extensor carpi radialis brevis muscles. The approach between the brachialis and triceps muscles can identify the radial nerve in the upper arm. The interval between the flexor carpi radialis and pronator teres approaches the median nerve.

A 35-year-old woman who is an artist is evaluated because of Boutonnière deformity with a flexed proximal interphalangeal (PIP) joint and an extended distal joint. She reports sustaining a laceration from a paint knife to the nondominant midline dorsal PIP joint 6 months ago. She did not seek medical attention at the time of injury. This patient's deformity is a result of attenuation of which of the following structures? A) Oblique retinacular B) Sagittal bands C) Terminal tendon D) Triangular ligament E) Volar plate

D A Boutonnière deformity occurs with injury to the central tendon and injury or attenuation of the triangular ligament. The lateral bands migrate volarly to become proximal interphalangeal (PIP) joint flexors, and their action is on the distal interphalangeal (DIP) joint, extending it. The triangular ligaments are bound by the lateral bands, central slip, and terminal tendon. The sagittal bands of the metacarpophalangeal (MCP) joint originate from the volar plate and anchor the extensor mechanism. The oblique retinacular ligament originates from the volar lateral crest of the proximal phalanx and inserts into the terminal tendon. The volar plates stabilize the MCP and PIP joints. The terminal tendon is the convergence to the lateral bands at the dorsum of the middle phalanx inserting on the distal phalanx.

A 22-year-old man who has consumed alcoholic beverages punches a concrete wall with both hands and sustains multiple metacarpal fractures. Which of the following will cause the most significant long-term hand impairment in this patient? A ) Fifth metacarpal neck fracture with 40-degree angulation B ) Fourth metacarpal neck fracture with 35-degree angulation C ) Fourth metacarpal shaft fracture with 10-degree angulation D ) Third metacarpal neck fracture with 25-degree angulation E ) Third metacarpal shaft fracture with 0-degree angulation

D Angulation is better compensated for in the ring and little fingers. The carpometacarpal (CMC) joints of these digits have 20 to 30 degrees of mobility in the sagittal plane. Angulation deformities in the little finger up to 40 to 70 degrees have been followed and found to have no functional impairment. However, because of the lack of compensatory CMC motion in the index and long metacarpal neck fractures, there is universal agreement that residual angulation greater than 10 to 15 degrees should not be accepted. Metacarpal shaft fractures generally require reduction for angulation greater than 30 degrees in the little finger, 20 degrees in the ring finger, and any angulation in the long and index fingers.

Consultation is requested for a 7-year-old girl because of intravenous infiltration of a chemotherapeutic agent in the dorsal forearm. Physical examination shows firmness and swelling of the forearm and pain on passive flexion of the wrist. Which of the following is the most appropriate initial management? A) Administration of an antidote B) Doppler sonography of the forearm C) Liposuction and saline flush of the affected area D) Measurement of compartment pressures E) Surgical excision and grafting

D Extravasation usually remains localized, yet some patients develop necrotic problem wounds. Often initially underestimated, the extent of injury can declare itself widely with time. Compartment syndrome in an extremity extravasation should be initially ruled out either by clinical assessment or direct measurement of compartment pressures. Tissue loss can include skin, muscle, tendon, nerve, vasculature, and/or joint. Given the variable amount of soft-tissue involvement, early conservative therapy is recommended. Immediate discontinuation of the infusion at the affected site is paramount and should not be overlooked. Aspiration, liposuction, wound excision, debridement, grafts, flaps, and antidote administration have all been described in the management of extravasation injury.

A 45-year-old man is brought to the emergency department immediately after sustaining injuries to the right upper extremity during a motorcycle collision. On admission, physical examination shows a flaccid and insensate right upper extremity, and x-ray studies show no abnormalities. Follow-up nerve conduction studies 4 weeks later show preservation of sensory nerve action potentials and evidence of fibrillations and denervation in the motor action potentials from the paravertebral muscles, biceps, triceps, and deltoid. The nerve injury is most likely located at which of the following levels of the nerve? A ) Anterior division B ) C5 and C6 trunk C ) Lateral cord D ) Preganglionic root E ) Subscapular nerve

D Following a traction injury to the brachial plexus, the nerves may rupture, be avulsed at the level of the spinal cord, or significantly stretch but remain intact. There are five possible levels where the nerve can be injured: (1) root, (2) anterior branches of the spinal nerves, (3) trunk, (4) cord, and (5) peripheral nerve. Root injuries may be further localized with respect to the dorsal root ganglion. Infraganglionic (postganglionic) injuries are located distal to the dorsal root ganglion, and supraganglionic (preganglionic) lesions are located proximal to the ganglion. With both types of lesions, patients will have the symptom of loss of muscle function. In supraganglionic injuries, the nerve has been avulsed from the spinal cord, separating the motor nerve fibers from the motor cell bodies in the anterior horn cells. The sensory fibers and cell bodies are still connected at the dorsal root ganglion; however, the efferent fibers entering the dorsal spinal column have been disrupted. Thus, sensory nerve action potentials will be preserved in patients with supraganglionic injuries, while motor nerve action potentials will be absent. In infraganglionic injuries, both the motor and sensory nerve cells have been disrupted, so there will be abnormalities in both motor and sensory action potentials. An injury to the C5 and C6 roots would show preservation of triceps function, as would an injury to the medial or lateral cord.

A 30-year-old man is brought to the emergency department after a rollover motor vehicle collision. Physical examination shows significant soft-tissue loss and a median nerve injury. At the time of surgical repair, there is a 5-cm gap in the nerve. Which of the following treatment options is most likely to provide the best long-term functional outcome for this patient? ` A) Acellular autograft B) Autologous vein graft C) Collagen conduit D) Peripheral nerve autograft E) Polyglycolic acid conduit

D For bridging long nerve gaps not amenable to primary repair (greater than 3 cm), peripheral nerve autografts are the most reliable choice. Their use is limited by supply and donor site morbidity from additional incisions, loss of sensation, and possible neuromas. Common donor sites include sural nerve, medial antebrachial cutaneous nerve, lateral antebrachial cutaneous nerve, dorsal cutaneous branch of the ulnar nerve, superficial peroneal nerve, and posterior and lateral cutaneous nerves of the thigh. Nerve conduits, such as silicone tubes, synthetic biodegradable tubes (such as polyglycolic acid conduit or collagen conduit), and biologic tubes (such as autologous vein grafts) are limited to use in short gaps (less than 3 cm). Beyond 3 cm, there is no clinically meaningful regeneration. Acellular autografts have recently been used with good success in short nerve gaps (less than 3 cm). The advantage of these products is that they provide the extracellular matrix molecules, which may enhance nerve regeneration.

A 45-year-old man who is a cyclist comes to the office because of tingling of the left ring and small fingers. Normal sensibility dorsally is noted. Guyon canal release is planned. Which of the following structures is a border of the Guyon canal? A) Capitate B) Dorsal distal radio-ulnar ligament C) Lunate D) Pisohamate ligament E) Volar distal radio-ulnar ligament

D Guyon canal, also known as ulnar canal and ulnar tunnel, allows passage of the ulna nerve and artery into the hand. It is a semi-rigid fibrosseous longitudinal tunnel, approximately 4 cm in length, beginning at the proximal edge of the transverse carpal ligament and ending at the hypothenar aponeurotic arch. The roof comprises the volar carpal ligament. The medial wall is the pisiform and pisohamate ligament. The lateral wall is the hook of the hamate. The floor is the flexor retinaculum and hypothenar muscles. The volar and dorsal distal radio-ulnar ligaments are the thicker portions of the triangular fibrocartilage complex (TFCC) and do not contribute to Guyon canal. The capitate and lunate are part of the floor of the carpal tunnel and also do not contribute to Guyon canal.

A 65-year-old man has a 4-cm defect in the mid portion of the metacarpal of the long finger after sustaining a gunshot wound to the left hand. Which of the following is the most appropriate treatment of this defect? A ) Coverage with a free vascularized bone flap B ) Distraction osteogenesis C ) Injection of calcium phosphate cement D ) Interpose an autologous bone graft E ) Placement of demineralized bone matrix

D In a patient with a noncritical (less than 6- to 8-cm) bone defect of the hand, reconstruction with an autologous bone graft provides the best combination of maximal healing and minimal morbidity. To provide the best chance of successful healing, the graft should allow rigid stabilization. Corticocancellous grafts from the iliac crest are the typical source. Free vascularized bone flaps, such as the fibula, are essential tools when dealing with critical bone defects (greater than 6 to 8 cm). They do, however, add significant morbidity to the procedure and may not be feasible in individuals with severe peripheral vascular disease. In this patient, the noncritical defect would argue against the need for vascularized bone. Distraction osteogenesis works well for bone defects ranging from 1.5 to 13.5 cm. A prerequisite, however, is adequate bone stock to allow pin placement for the distractor. In this case, a 4-cm defect would leave insufficient bone at the metacarpal base and head to allow distractor placement. Calcium phosphate cement (Norian, Synthes) offers an osteoconductive substrate in bone defects that are already stabilized. In addition, its use is contraindicated in infected or potentially infected wounds. Demineralized bone matrix is osteoinductive, but, like calcium phosphate cement, will not offer any structural stability.

A 42-year-old right-hand-dominant construction worker is evaluated for an 8-week history of pain with wrist motion. Physical examination shows swelling 4 cm proximal to the Lister tubercle. There is tenderness to palpation and crepitation over the muscle bellies of the abductor pollicis longus and extensor pollicis brevis. The remainder of the examination shows no abnormalities. Which of the following is the most likely diagnosis? A) Basal joint synovitis B) de Quervain synovitis C) Extensor pollicis longus tendinitis D) Intersection syndrome E) Wartenberg syndrome

D Intersection syndrome is synovitis of the second dorsal compartment. It is located where the abductor pollicis longus and extensor pollicis brevis cross the extensor carpi radialis longus and extensor carpi radialis brevis. The syndrome often occurs in athletes with repetitive forceful extension. The swelling is located 4 to 6 cm proximal to Lister tubercle. Basal joint arthritis and synovitis would have tenderness located at the wrist crease at the carpometacarpal joint and may have x-ray findings. de Quervain is synovitis of the first dorsal compartment, which would be located over the radial styloid and have a positive Finkelstein sign. Wartenberg syndrome is radial sensory nerve compression where the nerve exits the supinator muscle. Symptoms would be paresthesia over the radial nerve distribution and a Tinel sign. Extensor pollicis longus tendinitis is synovitis of the third dorsal compartment with vague dorsal wrist pain and tenderness, usually over the Lister tubercle with exacerbation with thumb extension.

A 30-year-old woman comes to the office because of a laceration of the dorsal long finger extensor tendon. Physical examination shows extension of all fingers to zero degrees at the metacarpophalangeal joint. Which of the following structures provides extension force that explains this exam finding? A) Deep transverse metacarpal ligament B) Intact paratenon C) Interosseous muscles D) Juncturae tendinum E) Lumbrical muscles

D Juncturae tendinum are tendon-like bands that connect the long, ring, and little finger extensor digitorum communis tendons. If the long finger extensor tendon is lacerated proximal to the attachment of the juncturae tendinum between the long and ring fingers, the ring finger extensor digitorum communis tendon will apply extension force to the distal long finger extensor tendon via the juncturae tendinum and produce metacarpophalangeal (MCP) extension. Interossesous and lumbrical muscles produce flexion of the MCP joint. The deep transverse metacarpal ligament stabilized the metacarpal heads relative to each other but does not produce motion at the MCP joint. In this patient, the tendon is visible and noted to be completely lacerated; therefore, the paratenon is not intact.

A 30-year-old man is evaluated for a hand injury after punching a wall. X-ray study shows a fracture of the fifth metacarpal neck. Which of the following findings is most likely to require surgical intervention? A) Angulation B) Callus formation C) Impaction D) Malrotation E) Shortening

D Malrotation causes scissoring of the affected digit, which will adversely affect neighboring digits. This will affect activities of daily living and is an indication for operative intervention. Other indications for operative intervention include displaced intra-articular fractures, severe soft-tissue injury, unstable open fractures, segmental bone loss, and multiple fractures. Angulation is usually dorsal tip-oriented because of intrinsic and extrinsic muscle pull. A good rule of thumb is the 10, 20, 30, 40 for digits two, three, four, and five. The fourth and fifth digits have carpometacarpal joint mobility and can tolerate larger angulation. The fifth digit metacarpal may even tolerate up to 70 degrees of angulation as long as there is no extensor lag. Impaction can lead to shortening and/or angulation, which is tolerated more than rotational deformities. Similarly, shortening is well tolerated as long as there is no extensor lag. Nonoperative complications include aesthetic with loss of knuckle and possible pain in the palm from the metacarpal head. Callus formation is indicative of an old, healed fracture.

A 68-year-old woman is evaluated because of numbness and paresthesias of the right hand. The diagnosis of carpal tunnel syndrome is confirmed by electromyography and nerve conduction velocity studies. The patient is interested in a trial of nonsurgical management. Nighttime splint immobilization and corticosteroid injection therapy to the carpal tunnel are planned. Which of the following best describes the most likely long-term outcome of this management strategy? A) Complete resolution of the patient's symptoms B) No effect on the patient's symptoms C) No initial effect on the patient's symptoms followed by gradual improvement over 6 to 12 months D) Short-term improvement of the patient's symptoms followed by a recurrence in 6 to 12 months E) Worsening of the patient's sensory and motor symptoms

D Nonsurgical management of carpal tunnel syndrome has been shown to be effective for symptomatic relief of carpal tunnel syndrome for up to 3 to 6 months, depending on the type of intervention that is chosen. Most patients who choose nonsurgical management will ultimately require surgery for carpal tunnel syndrome. Studies suggest that up to 70% of patients will go on to surgical carpal tunnel release at 1 year. Various nonsurgical interventions have been recommended for carpal tunnel syndrome. These include splint immobilization, oral corticosteroid therapy, local corticosteroid injection, ultrasonography, acupuncture, nonsteroidal anti-inflammatory drug (NSAID) therapy, and more. Many of the interventions are not supported by high-level evidence. Studies have shown improvement in carpal tunnel syndrome with splint immobilization, oral corticosteroid therapy, local corticosteroid injections, and ultrasonography when compared with a placebo or no treatment. Local corticosteroid injection into the carpal tunnel is more effective than oral corticosteroid therapy at 1 and 3 months without the potential risks of systemic steroid therapy. Splint immobilization plus a cortisone injection is more effective than splint immobilization only at 6 months. Full-time splint immobilization has not shown any benefit when compared with nighttime splint immobilization only. In patients who have mild to moderate carpal tunnel syndrome without static numbness, weakness, or thenar atrophy, short-term improvement or resolution of symptoms can be expected but long-term resolution is not likely. Patients with severe disease and signs of longstanding nerve compression including numbness and loss of abductor pollicis brevis strength may not respond to nonsurgical interventions. Worsening of symptoms as a result of nonsurgical treatment is unlikely. Carpal tunnel release has been shown to be more effective than splint immobilization and corticosteroid injections for symptomatic relief at 3, 6, and 12 months. However, many surgeons will offer a trial of nonsurgical management based on patient preferences. Complete resolution of symptoms following nonsurgical intervention is thought to be a good prognostic indicator for the success of surgery.

A 35-year-old woman who underwent surgical release of the first dorsal compartment 8 months ago is evaluated because of severe pain over the anatomical snuffbox of the right hand since the surgery. Physical examination shows focal tenderness over the region of the first dorsal compartment, a Tinel sign at the surgical scar, and numbness distal to the surgical scar. Medication includes gabapentin since the pain began, and the patient has attempted desensitization in hand therapy. Which of the following is the most appropriate next step in management? A) Exploration silicone capping of the palmar cutaneous nerve B) Initiation of oral pregabalin therapy C) Iontophoresis with dexamethasone D) Neurolysis and intramuscular placement of the superficial radial nerve E) Phenol injection into the scar and compression glove therapy

D One potential complication from surgical treatment of de Quervain disease (first dorsal compartment release) is injury to branches of the radial sensory nerve. When such injury is suspected, treatment should consist of exploration and, if possible, repair of the injured nerve branch. When this is not possible, multiple surgical options are available; transposition of the injured radial sensory nerve into the brachioradialis has been one of the commonly employed treatment strategies. Alternatives include transposition of the nerve into bone or a vein, nerve stripping, and coverage of the injured nerve with vascularized tissue. Both gabapentin and pregabalin are useful medical interventions for nerve pain. General guidelines suggest proceeding with surgical treatment for suspected neuromas if no improvement occurs after 6 months of oral therapy. Iontophoresis with corticosteroids has no proven efficacy for neuroma therapy. Phenol has been employed for stump neuromas and for Morton's neuromas; however, there is no evidence to support its use in non-amputation neuroma treatment. Silicone capping, once advocated for treatment of end-neuromas, has fallen out of favor because of poor pain relief results and displacement/migration of the caps.

A 35-year-old handyman comes to the office for follow-up 12 days after he underwent repair of the flexor digitorum profundus and superficialis (FDP and FDS) tendons of the little finger of the nondominant hand because of a knife injury. Early active motion was initiated during occupational therapy with sudden loss of flexion of distal and proximal joints yesterday. Which of the following is the most appropriate treatment? A) Delay treatment for 10 weeks, then place a silicone rod B) Discontinue occupational therapy for 2 weeks, then resume with a Duran passive protocol C) Fuse the proximal and distal joints of the little finger D) Repair the FDS and FDP tendons E) Transfer the FDS tendon of the ring finger to the FDP of the little finger

D Tendon rupture can occur early or late, up to 6 to 7 weeks, with days 7 to 10 being most common. Reoperation with repair of the previously repaired tendons will yield results similar to primary repair. Therefore, discontinuation of therapy and rod placement are incorrect. Repair should be undertaken before 2 weeks due to tendon shortening. Repair of both tendons will retain independent finger motion with greater power and decreased chance of proximal interphalangeal joint hyperextension. There will also be a better bed for FDP gliding. Therefore, flexor digitorum superficialis transfer is incorrect. Arthrodesis is reserved for failed treatment.

A 26-year-old man comes to the office 4 weeks after injuring his left shoulder while snowboarding. Physical examination shows limited abduction and forward flexion of the shoulder to 30 degrees. No additional abnormalities are noted. From which of the following areas of the brachial plexus does the affected nerve most likely arise? A) Lateral cord B) Lower trunk C) Medial cord D) Posterior cord

D The axillary nerve (ventral rami of C5 and C6) arises from the posterior cord of the brachial plexus, giving off muscular branches to teres minor and deltoid. It also innervates the shoulder joint and the skin over the deltoid. Its close proximity to the inferior shoulder capsule as it courses on the anteroinferior border of the subscapularis and then through the quadrangular space, puts it at risk for injury. The axillary nerve is most commonly injured during orthopedic surgeries such as shoulder arthroscopy, and open reduction and internal fixation (ORIF) of the proximal humerus, in which case, it is most commonly the result of closed traction injury. It can also be seen in the setting of an anterior glenohumeral joint dislocation or proximal humerus fracture, or as the result of a direct blow to the superior aspect of the shoulder. The majority of nerve injuries are temporary neurapraxias, which typically resolve within 6 to 12 months of injury; however, permanent nerve deficit can occur, requiring surgical intervention in the form of decompression, or reconstruction with nerve graft or nerve transfer from the radial nerve. The lateral cord receives contribution from C5, C6, and C7 roots, and contributes to the musculocutaneous and median nerves. The medial cord receives contribution from C8 and T1 roots, and contributes to the median and ulnar nerves. There are no superior or anterior cords within the brachial plexus.

A 53-year-old woman is evaluated for a 1-year history of numbness and tingling of the thumb and the index and long fingers of the right hand. She has been unresponsive to conservative treatment. An increase in which of the following is most likely suggestive of carpal tunnel syndrome in this patient? A) Abductor digiti minimi fibrillations B) Adductor pollicis fibrillations C) Motor nerve conduction velocity D) Sensory distal latency E) Sensory nerve conduction velocity

D The diagnosis of carpal tunnel syndrome is primarily a clinical diagnosis; however, electrodiagnostic studies (EDX) may be helpful in confirming the diagnosis. While these EDX studies are commonly referred to as "EMGs,"' they are actually two separate studies: the nerve conduction studies (NCS) and the electromyography (EMG). NCS examine both the sensory and the motor nerve fibers. Sensory nerve conduction studies measure sensory nerve action potential, and the motor nerve conduction studies evaluate a compound muscle action potential. The NCS also measures the amplitude of both the compound muscle action potential and sensory nerve action potential. Nerve conduction velocity (NCV), the velocity of the nerve's action potential between two points, is also measured by the nerve conduction studies. The EMG tests the muscle itself. The needle electrode examination can measure motor unit potential (MUP). MUP is measured in regards to its amplitude, duration, wave shape, and firing pattern. In the diagnosis of carpal tunnel syndrome, particular attention is given to the MUP of the abductor pollicis brevis muscle, which is uniquely innervated by the median nerve after it passes through the carpal tunnel. In the diagnosis of carpal tunnel syndrome, changes in the sensory nerve are detected earlier in the carpal tunnel process than motor changes. Early NCS changes (as compared to standardized normal values) include prolonged or increased sensory distal latencies. Prolonged motor latencies (also abnormal) are detected less frequently than the sensory latency changes, and detected in only 35 to 50% of patients with carpal tunnel syndrome. Motor amplitude change, found in carpal tunnel syndrome, is detected even less commonly. A conduction block, or slowing of the nerve's action potential velocity (NCV) between two points, can be seen with carpal tunnel syndrome. The needle electrode examination (EMG) is normal in more than 60% of patients with the diagnosis of carpal tunnel syndrome. Fibrillations in the abductor pollicis brevis occurs in generally less than 20% of patients with carpal tunnel syndrome. The adductor pollicis brevis and abductor digiti minimi muscles are innervated by the ulnar nerve, and would not show any electrodiagnostic evidence of muscle instability in isolated carpal tunnel syndrome.

Resistance to which of the following maneuvers is most likely present in a digit that has intrinsic tightness? A) Passive extension of the metacarpophalangeal (MCP) joint with the proximal interphalangeal (PIP) joint held in hyperextension B) Passive extension of the PIP joint with the MCP joint held in hyperflexion C) Passive flexion of the DIP joint with the PIP joint held in hyperextension D) Passive flexion of the PIP joint with the MCP joint held in hyperextension E) Passive flexion of the PIP joint with the MCP joint held in hyperflexion

D The intrinsic muscles (dorsal/palmar interossei and lumbricals) are responsible for much of the fine motor function of the hand. Contractures of these muscles lead to a loss of the delicate and complex balance of the intrinsic and extrinsic muscles and typically results in the clinical picture of an intrinsic-plus hand. The intrinsics attach to the extensor mechanism through the lateral bands and facilitate force transmission from the muscles to the extensor mechanism on the proximal and distal phalanges. Because of their line of pull, the intrinsics are responsible for metacarpophalangeal (MCP) joint flexion and proximal interphalangeal (PIP) joint extension. The intrinsic tightness test (i.e. Bunnell test) requires one to assess passive PIP joint flexion with the MCP joint extended. This is compared with passive PIP joint flexion with the MCP joint in flexion which assesses for extrinsic tightness. If there is a substantial increase in resistance to PIP joint flexion with the MCP joint in extension, then the test is considered positive and indicative of intrinsic tightness or adhesions of the lateral bands. Trauma is the most common cause of intrinsic muscle contracture. Spasticity from an upper motor neuron lesion (e.g. traumatic brain injury, cerebrovascular accident, cerebral palsy) may also lead to intrinsic contracture. Arthritis may also lead to intrinsic contracture resulting from joint deviation or dislocation. In trauma, initial treatment is directed at edema prevention and aggressive hand therapy. Patients with spasticity from an upper motor neuron disorder are also initially managed with therapy and splinting. If these modalities are insufficient, surgical release of the intrinsic muscles or tendons (proximal or distal depending on extent of involvement) may improve posture and function. Ulnar motor neurectomy is another option in severely affected individuals to decrease intrinsic muscle tone and improve posture and function, but is only effective in the absence of a fixed MCP joint contracture.

A 23-year-old man comes to the office for follow-up evaluation 14 months after sustaining a closed brachial plexus injury in a motor vehicle collision. He was initially treated at another facility with occupational therapy and observation. He has been compliant with therapy. Physical examination shows 4/5 strength in shoulder abduction, elbow flexion, elbow extension, and finger flexion. He is unable to extend the wrist or fingers but has good passive mobility of the wrist and fingers. Which of the following is the most appropriate next step to restore wrist and finger extension? A) Distal nerve transfer B) Intraplexus nerve grafting C) Pedicled latissimus muscle transfer D) Tendon transfer E) Continued observation

D The most appropriate management for the patient described is tendon transfers to restore wrist, finger, and thumb extension. Following closed brachial plexus injuries, patients should receive CT scan or MR myelogram and electrodiagnostic studies at 3 to 4 weeks. This will allow enough time to see pseudomeningoceles and denervation changes. The electromyography and nerve conduction studies are generally repeated at 14 to 16 weeks to look for signs of regenerating axons, and this information is used to help determine the strategy for reconstruction. Surgery is recommended in the absence of clinical or electrical evidence of recovery. The patient described has late symptoms, and any strategy that involves attempting to repair or reconstruct the injured nerves is not recommended. After 12 to 18 months, useful motor recovery is unlikely due to intraneural fibrosis, loss of Schwann cells, muscle atrophy, and motor end-plate degeneration. Tendon transfers may be done at any time assuming that there are suitable donor tendons (at least 4/5 strength and full excursion) and that full passive mobility is present. In this example, the patient has adequate donor tendons from the median and ulnar nerves, and full passive range of motion. Examples of tendon transfers include: palmaris longus to extensor pollicis longus, pronator teres to extensor carpi radialis brevis, and flexor carpi ulnaris to extensor digitorum communis. Early or subacute exploration of the injured plexus with resection and intraplexal nerve grafting can be used with ruptures or neuromas that do not conduct a nerve action potental across the lesion. In adults, grafting is reserved for C5, C6, and C7 to restore shoulder abduction, elbow flexion, elbow extension, and wrist extension. Nerve grafting for lower trunk injuries in adults is generally not successful due to the length and time required for the regenerating axon to reach the distal target muscles. Nerve transfers have become a very useful and versatile tool for reconstruction of brachial plexus and peripheral nerve injuries. A less important distal nerve is sacrificed to replace the function of a more important nerve. Nerve transfers can be performed in preganglionic injuries and to decrease the distance to the target muscle for reinnervation. Ideally, nerve transfers are performed within 6 months of the injury. Studies have shown greater than 70% will achieve M3 function for elbow flexion and shoulder abduction. Common donor nerves include the spinal accessory, intercostals, anterior interosseous, and triceps branch. Other options for late reconstruction include pedicled muscle transfers and neurotized functional free muscle transfers. The latissimus dorsi muscle can be used to restore elbow flexion or elbow extension but will not reach beyond the elbow. Currently, free muscle transfer is the best option to restore hand and wrist function in complete brachial plexus palsy.

A 47-year-old man undergoes repair of a laceration to the extensor tendon of the long finger at the dorsum of the left hand. He wishes to restore function of the hand as quickly as possible. Which of the following is the most appropriate course of splint immobilization? A) Continuous extension splint immobilization for 4 weeks B) Continuous extension splint immobilization for 6 weeks C) Dynamic extension splint immobilization for 6 weeks D) ative motion extension splint immobilization for 6 weeks E) No immobilization

D The most appropriate treatment is relative motion extension splint immobilization for 6 weeks. The relative motion extension splint allows immediate controlled active motion. Placing the injured tendon in 15 to 20 degrees less motion than the adjacent tendons results in significantly less force. A splint is fashioned placing the repaired extensor tendon of the long finger in 15 to 20 degrees more metacarpophalangeal extension than the neighboring extensor tendons. Full interphalangeal joint range of motion is permitted. The finger splint is worn with a wrist component for the first 3 weeks (which may not be necessary). The finger component is used for only 3 additional weeks. This allows for earlier recovery of motion and return to work. Without immobilization, the repair is at increased risk for rupture and failure. Extensor tendon injuries have typically been treated with continuous extension splint immobilization for 4 to 6 weeks. Although this would be a reasonable option, use of the relative motion extension splint allows immediate movement and decreases the stiffness that may result from immobilization. This may be preferred in children or noncompliant patients. Dynamic extension splinting is a possibility and may be useful in patients who have an extensor pollicis longus injury or in patients in whom all six finger extensors are severed. Relative motion splinting is not an option in these cases.

A 48-year-old woman has numbness and paresthesia of the right hand 2 years after mastectomy and radiation therapy. She reports no pain or night waking. Symptoms have not improved with cock-up wrist splints or injection of a corticosteroid into the carpal tunnel. Physical examination shows swelling and weakness of the right arm, most prominently in the C5-C6 distribution; no varicosities, stasis ulcers, dermatitis, or symptoms of Horner syndrome are noted. Allen test is normal. Electromyography shows myokymia. CT scan shows diffuse swelling but no mass. Which of the following is the most likely diagnosis? A ) Acute ischemic injury B ) Carpal tunnel syndrome C ) Chronic venous insufficiency D ) Radiation-induced brachial plexopathy E ) Tumor recurrence

D The most likely diagnosis is radiation-induced brachial plexopathy, which can occur when radiation therapy is directed at the chest, axillary region, thoracic outlet, or neck. The incidence is 1.8 to 4.9% of those patients receiving radiation therapy to the above areas and is most common in patients with underlying breast or lung carcinoma. Patients often have sensory symptoms, with swelling and a generalized weakness of the arm. Eighteen percent of patients have pain in the shoulder, wrist, or hand. The neurologic findings are most prominent in the C5-C6 distribution. The lymphatic-vascular system may show prominent lymphedema of the involved extremity without cyanotic or dusky features. There should be no disturbance of arterial or venous circulation in the involved extremity and no changes in the limb to suggest venous insufficiency (ie, varicosities, stasis ulcers, or dermatitis). The Allen test should be normal. Horner syndrome is not present in patients with radiation-induced brachial plexopathy. Eighty percent of patients with tumor infiltration into the brachial plexus come to the office because of pain in the shoulder, upper arm, elbow, and ring and little fingers. Symptoms progress to atrophy and weakness of the C7-T1 distribution with persistent pain and occasional Horner syndrome. CT scan shows a discrete mass with circumscribed tissue infiltration. Electromyography shows segmental slowing. Patients with acute ischemic injury have symptoms of paresthesia in the C5-C6 nerve distribution and acute, nonprogressive weakness and sensory loss. CT angiography would demonstrate subclavian artery segmental obstruction. Electromyography shows segmental slowing. Patients with carpal tunnel syndrome often have night waking and experience a period of symptomatic relief after injection of a corticosteroid into the carpal tunnel. Chronic venous insufficiency does not typically have neurologic sequela.

A 34-year-old man is brought to the emergency department 2 hours after sustaining injuries to the right wrist when he punched a glass window. Surgical exploration shows a complete laceration of the median nerve at the level of the wrist. A 1-cm gap between the proximal and the distal stumps of the nerve is noted. Which of the following treatments is most likely to provide the best functional outcome? A) Multistrand nerve grafting B) Nerve transfer C) Nerve transposition D) Primary epineurial repair E) Single-strand nerve grafting

D The need for nerve grafting is dependent upon many parameters, such as the length of the gap, the excursion of the nerve, the wound bed, and vascularity, among others. For clean, sharp injuries with nerve gaps measuring less than 1 cm in a large peripheral nerve such as the median, most authors agree that primary repair of the nerve results in the best outcome. Autologous nerve grafting should be reserved for cases in which there is tension on the nerve ends with primary repair. Both multistrand nerve grafting and single-strand nerve grafting produce similar outcomes and are inferior to primary repair. Nerve transfer would only be considered if there were no proximal nerve to repair to the distal nerve. Nerve transposition would only be appropriate for gaining length in the ulnar nerve, where the switch from the extensor side of the elbow to the flexor side results in increased relative length in the nerve.

A 36-year-old man comes to the office because of a 16-month history of diffuse pain of the posterior right shoulder. There is no history of trauma. Results of x-ray study are negative for an osseous injury. Physical examination shows atrophy isolated to the posterior scapular muscles. Motor and sensory examination of the right upper extremity shows weakness in shoulder external rotation with the arm adducted; no other abnormalities are noted. Which of the following nerves is most likely injured? A) Axillary B) Long thoracic C) Spinal accessory D) Suprascapular E) Thoracodorsal

D The patient has an isolated palsy of the suprascapular nerve, the first branch off of the upper truck (C5, C6) of the brachial plexus. Causes can include trauma, ganglion cyst (supraspinous fossa), or direct compression of the nerve as it passes under the transverse scapular ligament. Symptoms include diffuse posterior shoulder pain, atrophy of the supraspinatus and infraspinatus muscles, and weakness in shoulder external rotation. The axillary nerve comes off of the posterior cord and innervates the deltoid muscle and teres minor muscle; palsy would primarily impair shoulder abduction. The long thoracic nerve is composed of contributions from the C5-C7 roots and innervates the serratus anterior muscle; a deficit would lead to scapular winging. The thoracodorsal nerve (C6-C8), a branch of the posterior cord, innervates the latissimus dorsi muscle; a deficit would impair shoulder extension, adduction, and internal rotation. The spinal accessory nerve innervates the trapezius and sternocleidomastoid muscles. Injury to this nerve does not affect the periscapular musculature.

A 26-year-old man sustained a crush injury to the tip of the left middle finger with an associated fracture at the dorsal base of the distal phalanx with nail bed injury 6 months ago. No treatment was provided. Examination shows non-union of the distal phalanx. Which of the following is the most likely secondary deformity in this patient? A) Boutonniere deformity B) Jersey finger C) Quadriga D) Swan neck deformity E) Trigger finger

D The scenario described involves a bony mallet deformity in which a distal phalanx fracture is associated with disruption of terminal extension at the distal interphalangeal joint. If untreated, the DIP extension loss due to a non-union of a bony mallet injury may progress to a swan neck deformity through compensatory proximal phalangeal hyperextension in the setting of continued and persistent flexion at the distal interphalangeal joint (from unopposed pull of the flexor digitorum profundus tendon). A secondary swan neck deformity may occur because of dorsal subluxation of the lateral bands and attenuation of the volar plate and transverse retinacular ligament at the PIP joint level. A jersey finger is caused by rupture of the terminal flexor digitorum profundus. A boutonniere deformity can be caused by an injury to the central slip (but not the terminal extensor tendon). Quadriga is due to loss of length of a repaired FDP tendon, causing the finger with the repaired tendon to reach terminal flexion sooner than the other fingers whose FDP tendons are of normal length. A trigger finger does not involve a fracture of the DIP joint.

A 53-year-old man is evaluated because of a 5-month history of numbness and tingling of the right ring and little fingers. He says that symptoms are worse at night and when his elbow is flexed while holding a phone to his ear. Physical examination shows weakness of abduction of the fingers of the right hand. Which of the following structures is the most likely cause of this patient's symptoms? A ) Arcade of Frohse B ) Flexor digitorum superficialis C ) Ligament of Struthers D ) Osborne band E ) Transverse carpal ligament

D The structure most likely to be responsible for the condition described is Osborne band. The patient described exhibits symptoms of ulnar nerve compression at the cubital tunnel. Cubital tunnel syndrome is characterized by numbness and tingling of the ulnar nerve distribution (ulnar side of hand, involving little finger and ulnar half of ring finger) and can lead to intrinsic weakness. The ulnar nerve passes posterior to the medial epicondyle and travels between the medial epicondyle and the olecranon at the level of the elbow. In the region of the cubital tunnel, a set of thick fascial fibers known as Osborne band or ligament forms the roof of the tunnel and can compress the ulnar nerve. A number of additional sites can contribute to ulnar nerve compression, including the arcade of Struthers, the medial intermuscular septum, anconeus epitrochlearis, and the fascia of the flexor carpi ulnaris or flexor pronator musculature. The arcade of Frohse is a fibrous arch that comes from the lateral epicondyle, which can compress the radial nerve. The flexor digitorum superficialis can cause compression of the median nerve in the forearm. The ligament of Struthers is an anatomic variant which arises from the supracondylar process of the humerus. It can cause proximal median nerve compression. The transverse carpal ligament is responsible for median nerve compression at the carpal tunnel.

A 55-year-old man with bilateral carpal tunnel syndrome comes to the outpatient surgical unit for elective surgical intervention of the dominant right hand. He will be the tenth procedure of the day for the surgeon performing the operation. The surgeon favors an open technique; he has performed 150 carpal tunnel operations since finishing his hand fellowship 3 years ago. Which of the following is most likely to increase the risk of wrong-site surgery? A) The elective nature of the procedure B) Only one surgeon is involved in the operation C) The procedure will be performed using an open technique D) The surgeon has a high volume of cases scheduled for the same day E) The surgeon has only been practicing independently for 3 years

D The term ?wrong-site surgery? includes surgery on the wrong organ or extremity, the wrong patient, or the wrong vertebral level. This error can result in disastrous outcomes for the patient, as well as the institution and professionals involved. Traditionally, these errors have been considered ?sentinel events? that require a root cause analysis to define the hazards that triggered the event. Fortunately, wrong-site surgery is rare; however, the true incidence is unknown and appears to be increasing. Attempts to quantify the true incidence of wrong-site surgery are limited by underreporting to The Joint Commission and the often covert nature of these events caused by liability concerns. One review has estimated an incidence rate of one wrong-site surgery per 100,000 operations. This rate was 4 times higher among hand surgeons, however, with an estimated one in five hand surgeons predicted to perform a wrong-site surgery in their career. A Joint Commission review of a series of sentinel events identified a number of factors contributing to the increased risk of wrong-site surgery, such as emergency cases; unusual physical characteristics, including morbid obesity or physical deformity; unusual time pressures to start or complete the procedure; unusual equipment or setup in the operating room; multiple surgeons involved in the case; and multiple procedures being performed during a single surgical visit. A large series of wrong-site hand surgeries showed an increased rate of wrong-site surgery with increasing surgeon age and experience, and a direct correlation with increasing surgical case volumes. While more than one factor was often identified after a root cause analysis, the majority involved a breakdown in communication between the surgical team and the patient and his or her family. While it seems that something as simple as operating on the correct side would be intuitive, in reality, the enormous pressures of time and patient volume in the current health care environment set up surgeons for an eventual system failure. Review of the factors noted above demonstrates that the major component in preventing wrong-site surgery is effective communication between surgeon and patient, and the surgeon bears the ultimate responsibility of assuring that this level of communication exists. Current Joint Commission requirements include a preoperative verification, site marking, and a ?time out? in the operating room. One recent study of malpractice claims after wrong-site surgery showed that two thirds of wrong-site surgeries could have been prevented by an effective site-verification protocol. Recommendations for effective site verification include the following:

A 24-year-old male figure skater comes to the emergency department because of dorsal metacarpophalangeal dislocation of the right thumb. The emergency department staff is unable to reduce the dislocation. Which of the following structures is most likely preventing reduction in this patient's injury? A) Extensor pollicis longus tendon B) Flexor pollicis brevis tendon C) Opponens pollicis D) Sesamoid bone E) Ulnar neurovascular bundle

D The thumb metacarpophalangeal (MCP) sesamoid bone(s) may be associated with an irreducible dorsal dislocation. Dorsal dislocation of the thumb at the MCP joint typically occurs with forced hyperextension with resultant volar plate and collateral ligament rupture. Irreducibility usually occurs through interposition of the volar plate in the joint. Extensor expansion interposition, collateral ligaments, bony fragments, sesamoids, and flexor pollicis longus entrapment have also been associated with irreducibility. In these circumstances, open reduction is often necessary. The ulnar neurovascular bundle, extensor pollicis longus, opponens pollicis, and flexor pollicis brevis are not typically associated with an irreducible dislocation.

A 35-year-old man comes to the emergency department with a humerus fracture. On examination, he is unable to extend his wrist, fingers, and thumb. Which of the following nerves is most likely injured? A) Axillary B) Median C) Musculocutaneous D) Radial E) Ulnar

D This patient has a radial nerve injury, which can occur with humerus fractures. The radial nerve innervates the wrist extensors, extensor carpi radialis longus (ECRL) and extensor carpi radialis brevis (ECRB); the thumb extensors, extensor pollicis longus (EPL) and extensor pollicis brevis (EPB); and the finger extensors, extensor digitorum communis (EDC). These radial nerve injuries are usually managed with a period of observation and therapy until a potential neurapraxia resolves. Early evidence of muscle reinnervation would be evident with improved function of the ECRL, followed by the ECRB, then the finger and thumb extensors. If the patient does not regain any function by 3 to 6 months, exploration and nerve repair or tendon transfers to restore lost function can be considered. The standard tendon transfers considered for radial nerve function loss include the pronator teres to the ECRB (wrist extension), flexor carpi ulnaris to the EDC (finger extension), and the palmaris longus to the EPL (for thumb extension). A median nerve deficit would result in loss of flexion of wrist, fingers, thumb, and loss of palmar sensation and not typical after humerus fractures. An ulnar nerve injury could present with loss of hand intrinsic function and loss of sensation of the small finger. The musculocutaneous nerve innervates the biceps and would result in loss of elbow flexion. The axillary nerve is not injured with humerus fractures.

An otherwise healthy 58-year-old man comes to the office because of numbness of all fingers of the left hand. He says he first noticed symptoms after a cross-country drive 6 weeks ago. On physical examination, the thumb, index, and long fingers show sensitivity to the 2.83 Semmes-Weinstein monofilament. The ring and little fingers show sensitivity to the 3.22 monofilament. The little finger is held in an abducted position. Abductor pollicis brevis muscle strength is normal. First dorsal interosseous muscle strength is diminished. Which of the following anatomical structures is the most likely cause of these findings? A) Arcade of Frohse B) Lacertus fibrosus C) Ligament of Struthers D) Osborne ligament E) Transverse carpal ligament

D This scenario depicts a patient with ulnar nerve compression. Often patients who present with compression neuropathies give a history of numbness of all fingers; however, careful physical examination will show sensory abnormalities only in the anatomical location of the compression. The patient has weakness of the ulnar nerve innervated intrinsic muscles, the first dorsal interosseous muscle, but retains strength in the abductor pollicis brevis muscle. The diagnosis of ulnar nerve compression is suggested. The most common sight of ulnar nerve compression is at the elbow. The anatomical causes of all the nerve compression at the elbow are the arcade of Struthers, the medial intramuscular septum, the bony cubital tunnel, Osborne ligament, an anconeus epitrochlearis muscle, and the origin of the flexor carpi ulnaris muscle. The ligament of Struthers, lacertus fibrosus and the transverse carpal ligament are anatomical sites of compression of the median nerve. The arcade of Frohse is a site of compression of the radial nerve.

A 23-year-old man is brought to the emergency department because of a laceration of all extensor tendons at Zone VII of the right upper extremity. Which of the following tendons has the most distal muscle belly when attempting to reappose the tendon ends? A) Extensor carpi radialis longus B) Extensor carpi ulnaris C) Extensor digitorum communis to long finger D) Extensor indicis proprius E) Extensor pollicis longus

D Zone 7 extensor tendon injuries are those over the dorsal wrist. The extensor indicis proprius tendon typically has the most distal muscle belly and this fact can frequently be used to uniquely identify this tendon. Extensor tendon zones are useful for describing the locations of injuries: Distal interphalangeal (DIP) Central slip to DIP Proximal interphalangeal (PIP) Metacarpophalangeal (MCP) to PIP MCP Carpometacarpal (CMC) to MCP Wrist and proximal

A 54-year-old right-hand-dominant man comes to the office because of a 1-year inability to fully extend the right thumb after sustaining a laceration. Medical history includes no abnormalities.The patient reports being unable to grasp large objects. Hand and wrist x-ray studies show no abnormalities. An extensor pollicis longus (EPL) tendon injury is suspected. Exploration is planned. Which of the following additional interventions is the most appropriate next step in management? A) Primary four-strand repair of the EPL tendon B) Repair of the EPL tendon with a palmaris longus tendon interposition graft C) Tenorrhaphy of the extensor pollicis brevis tendon and EPL tendon D) Transfer of the extensor indicis proprius to the EPL tendon E) Transfer of the palmaris longus tendon to the EPL tendon

D ruptures of the extensor pollicis longus (EPL) tendon. The EPL tendon is prone to rupture from synovitis and friction at Listers tubercle. Since these are identified months after the original injury, primary repair is not possible secondary to retraction of the tendon and/or atrophy of the tendon ends. The EIP is the preferred tendon for the transfer because it has an appropriate direction and excursion compared with the EPL. This tendon transfer has demonstrated excellent outcomes in previous studies. Tenorrhaphy of the EPL to the extensor pollicis brevis tendon would not allow full thumb extension. Given the chronicity of this injury, sufficient myostatic contracture has occurred to render tendon interposition grafting inferior to EIP tendon transfer.

Myoelectric prostheses offer which of the following advantages over body-powered prostheses? A) Higher durability B) Lower cost C) Lower frequency of adjustment D) More complex motions performed E) Shorter training time

D Body-powered prostheses have been shown to have advantages in durability, training time, frequency of adjustment, maintenance, and feedback; however, they could still benefit from improvements of control. Myoelectric prostheses have been shown to provide greater range of motion including more complex movements involving multiple joints moving at the same time. Currently, evidence is insufficient to conclude that either system provides a significant general advantage. Prosthetic selection should be based on a patient's individual needs and include personal preferences, prosthetic experience, and functional needs.

A 36-year-old man presents with weakness of the left wrist and limited finger extension 2 days after he underwent surgery for a closed fracture of the right humerus that he sustained in a motor vehicle collision. At the time of surgery, the radial nerve was found to be intact without any significant signs of trauma. On examination, the patient is unable to extend his wrist or digits but elbow extension is intact. Which of the following histopathologic features seen in neurapraxia (Sunderland Type 1) injury is most likely in this patient? A) Disruption of perineurium B) Endoneurial fibrosis C) Increased axonal transport D) Segmental demyelination E) Wallerian degeneration

D Peripheral nerve injuries can be classified into three main categories: Neurapraxia, axonotmesis, and neurotmesis. These groups have been further classified by several different classification systems, which include those by Seddon and Sunderland, among others. Neurapraxia (Sunderland type 1) is an injury to the myelin sheath only, while axons are preserved. In trauma, these injuries are most frequently caused by compression or stretching. Although segmental demyelination occurs (leading to conduction block), there is no Wallerian degeneration of the nerve, and a full recovery can be expected within days to weeks. Axonotmesis (Sunderland type 2, 3 or 4) involves damage to axons, and is characterized by Wallerian degeneration. Sunderland type 2 injury involves only the axons, and usually there will be a full recovery without intervention, while types 3 and 4 involve injury to the endoneurium and perineurium respectively and fail conservative management. Neurotmesis (Sunderland type 5) is a complete disruption of a peripheral nerve. MacKinnon and Dellon described a type 6 injury that involves mixed Sunderland type injuries along the length of a damaged nerve. Axonal transport is not a relevant histopathological feature.

A 52-year-old man presents for evaluation of a claw deformity of the right ring and small fingers. Medical history includes an unrepaired low ulnar nerve injury sustained 30 years ago. Which of the following is the most likely pathophysiology of this patient's deformity? A) Unbalanced abductor digit minimi muscle B) Unbalanced median and ulnar innervated intrinsic muscles C) Weak thenar muscles D) Weak ulnar innervated extrinsic flexor muscles E) Weak ulnar innervated intrinsic muscles

E Clawing after ulnar nerve injury includes hyperextension of the metacarpophalangeal (MCP) joints and flexion of the interphalangeal (IP) joints. The pathophysiology includes paralysis of the interossei and third and fourth lumbricals. Unopposed long extensors cause the metacarpophalangeal joints to fall into extension while the long flexors pull the proximal interphalangeal joints into flexion. This posture is the classical 'claw hand.'

A 30-year-old man is evaluated because he is unable to abduct or externally rotate his shoulder or flex his elbow 4 months after he was involved in a high-speed motor vehicle collision. Physical examination shows numbness of the lateral upper arm and forearm. Which of the following nerve transfers is most appropriate to restore external rotation of the shoulder? A) C7 ipsilateral root to anterior division of upper trunk B) Medial pectoral nerve to medial cord C) Phrenic nerve to long thoracic nerve D) Radial nerve to axillary nerve E) Spinal accessory nerve to suprascapular nerve

E Examination findings indicate a C5-6 avulsion or very proximal upper trunk injury. External rotation of the shoulder is provided by the supraspinatus and infraspinatus muscles. Multiple techniques for harvest of the distal spinal accessory nerve to transfer to the suprascapular nerve have been described. This transfer would restore external rotation. Transfer of the radial nerve to the axillary nerve will provide deltoid and possibly teres major innervation. This will improve shoulder function overall but will not restore external rotation. The phrenic nerve is commonly used to provide donor motor axons, but the long thoracic nerve goes to the serratus anterior. Contralateral C7 transfer can be used to innervate the upper trunk. Ipsilateral C7 would not be used because the C5 and C6 roots were destroyed in the injury. Also, coaptation to the anterior division of the upper trunk would be distal to the origin of the suprascapular nerve. The medial pectoral nerve can also be used for donor motor axons. It is limited by its relatively short reach. Coaptation to the medial cord of the brachial plexus would innervate the ulnar nerve, which is not injured in this patient.

A 32-year-old man comes to the office because of a "mallet" deformity of the distal joint of the long finger of the dominant hand sustained 12 years ago while he was playing baseball. He has not sought medical treatment until now. On examination, which of the following deformities is most likely? A) Boutonnière B) Camptodactyly C) Clinodactyly D) Hook-nail E) Swan-neck

E Flexion deformity of the distal joint seen in mallet finger will lead to secondary hyperextension of the proximal joint. This occurs in a zigzag fashion because of the imbalance of forces. If the terminal tendon is displaced proximally, the conjoined tendons will slide proximally and become extensors to the proximal joint. A boutonnière deformity is a flexion deformity of the proximal joint from disruption of the central slip. The lateral slips migrate volarly becoming an extensor to the distal joint which then hyperextends. Clinodactyly is a genetic condition in which there is a curvature of the fifth finger toward the fourth finger. A hook nail usually results from loss of nail bed support, usually after amputation. Camptodactyly is also a genetic condition in which there is a fixed flexion deformity of the proximal joint of the little finger.

An 80-year-old woman comes for evaluation because she is unable to flex the tip of the little finger of her nondominant hand 9 months after sustaining a laceration from a knife. She did not seek treatment at the time of injury. She has no pain or any difficulty with activities of daily living. Physical examination shows a well-healed laceration over the volar aspect of the middle phalanx. Active range of motion is full in the metacarpophalangeal and proximal interphalangeal (PIP) joints; there is no flexion at the distal interphalangeal joint. Which of the following is the most appropriate management? A) Delayed primary flexor digitorum profundus repair B) Flexor digitorum superficialis transfer C) PIP joint arthrodesis D) Staged flexor tendon reconstruction with placement of a silicone rod followed by tendon grafting E) No intervention is indicated

E No intervention is indicated for this patient. The principles of tendon repair and reconstruction have evolved since the first description of primary tendon repair in Zone II in 1967. Proper patient selection is essential before attempting any reconstruction to restore functional motion. The indications for grafting or reconstructing through an intact flexor digitorum superficialis (FDS) are narrow, and sacrifice of an intact FDS is generally not recommended. Most of the functional arch of motion is maintained with the superficialis tendon, and many patients will function well with an FDS digit alone. Profundus reconstruction through an intact FDS is most often reserved for patients between 10 and 21 years old with high occupational demands for dexterity such as artists or musicians. This patient is beyond the recommended age range for an optimal outcome and is reporting no functional deficits as a result of her injury. In addition, delayed symptoms may give a clue to the patient's ability to comply with rigorous postoperative therapy. Delayed primary repair can be attempted up to several weeks after a flexor tendon injury, and up to 6 weeks in pediatric patients. However, 9 months is well beyond the time when a primary repair would be possible. The FDS from an adjacent finger can be used as the proximal motor in cases of tendon graft reconstruction or tendon repair rupture. This is a consideration if the native proximal stump of the profundus is significantly damaged or scarred and has poor excursion. In this patient, reconstruction is not indicated, and nothing is mentioned regarding the proximal tendon. A distal interphalangeal (DIP) joint arthrodesis may be indicated if the DIP joint is unstable with a well-functioning proximal interphalangeal (PIP) joint. Tenodesis of the flexor digitorum profundus stump is another option for soft-tissue DIP stabilization. Tendon reconstruction is indicated when a delay in treatment makes primary tendon repair impossible. A healed wound with full passive range of motion, absence of significant scarring, and an intact flexor retinacular pulley system are considered prerequisites for a single-stage reconstruction. In any other situation, a two-stage reconstruction with implantation of a temporary silicone rod is indicated. This would be the reconstructive strategy of choice for this patient if the FDS tendon were also involved in the original injury.

A 33-year-old woman comes to the office because of volar numbness of the right thumb, index, long finger, and palm for the past 4 months. Medical history includes no abnormalities. Physical examination shows weakness of the palmar abduction of the thumb and interphalangeal joint flexion of the thumb. Percussion tenderness over the volar aspect of the wrist does not reproduce symptoms. Passive flexion of the wrist does not reproduce symptoms. Which of the following is the most likely diagnosis? A) Anterior interosseous syndrome B) Carpal tunnel syndrome C) Cubital tunnel syndrome D) Posterior interosseous syndrome E) Pronator syndrome

E Pronator syndrome is a compression neuropathy of the median nerve in the proximal forearm. Paresthesias in the palm and the median nerve-innervated fingers of the thumb along with weakness of the flexor pollicis longus muscle are classic findings in pronator syndrome. Pronator syndrome can be differentiated from carpal tunnel syndrome by sensory symptoms and numbness occurring in the palm, which is innervated by the palmar cutaneous branch of the median nerve. This branch comes off the proper median nerve proximal to the carpal tunnel. Findings of weakness of the flexor pollicis longus muscle and often the flexor digitorum profundus muscle to the index finger are often seen in pronator syndrome. Anterior interosseous syndrome is a motor-only compression of the median nerve in the proximal forearm; sensory findings are not present. Cubital tunnel syndrome or ulnar nerve compression at the elbow presents with sensory findings in the small and ring fingers and may also affect the interosseous muscles. Posterior interosseous syndrome affects the extensor muscles in the forearm and is not the pathologic lesion described in the scenario.

A 37-year-old woman presents for evaluation of a laceration to the mid humerus that she sustained in a motor vehicle collision. On examination, the patient is unable to extend the wrist, fingers, or thumb. Surgical exploration shows complete radial nerve transection; the median/ulnar nerves are intact. Direct neurorrhaphy is performed after debridement and mobilization of the nerve ends. Which of the following is the last muscle to be reinnervated during nerve recovery? A) Abductor pollicis brevis B) Abductor pollicis longus C) Brachioradialis D) Extensor carpi radialis brevis E) Extensor indicis proprius

E Radial nerve injuries may occur in the setting of humeral fractures, and transection is most common in the setting of an open injury. The most important components of functional recovery following radial nerve injury include wrist, finger, and thumb extension. The order of reinnervation of the radial-innervated muscles is most commonly brachioradialis, extensor carpi radialis longus, supinator, extensor carpi radialis brevis, extensor digitorum communis, extensor carpi ulnaris, extensor digiti quinti, abductor pollicis longus, extensor pollicis longus, extensor pollicis brevis, and extensor indicis proprius. The abductor pollicis brevis is innervated by the median nerve via the thenar motor branch.

Which of the following is most likely an open fracture? A) Barton B) Colles C) Epiphyseal D) Salter-Harris III E) Seymour

E The Seymour fracture in children displaces through the epiphysis with the nail matrix interposed between the fragments. It is always an open fracture. The Salter-Harris classification is the most commonly used method to describe the five most frequent patterns of pediatric fractures involving the physis. The classification helps explain the mechanism of injury and anticipate the consequences of the fracture upon subsequent growth. A Salter-Harris III fracture is epiphyseal but is not necessarily open. Colles and Barton fractures are seen in the distal radius.

A 19-year-old college baseball player comes for evaluation 4 weeks after he jammed and dislocated the long finger of his dominant right hand while sliding into home plate. His coach reduced the dislocation on the field. The patient says he has noticed increasing pain at the site of the injury in the past two days. Physical examination shows edema of the proximal interphalangeal (PIP) joint of the long finger. Lateral stress shows angulation of 30 degrees. X-ray study shows a congruous joint with radial side widening of 1 mm. Which of the following is the most appropriate management? A) Buddy taping of the long finger to the index finger B) Immobilization in a dorsal extension block splint at 30 degrees C) Immobilization with a volar short arm splint D) Kirschner wire fixation of the PIP joint for 3 weeks E) Open repair of the radial collateral ligament

E The goal of treatment is to restore normal finger function; prevent pain, stiffness, and traumatic arthritis; and to restore activities of daily living. This patient has a complete tear of the radial collateral ligament of the proximal interphalangeal (PIP) joint of the long finger. Widening of the joint space indicates probable interposition of ligament fibers. Angulation greater than 20 degrees is associated with poor prognosis. Complete tears with subluxation and interposition require surgical repair.Most collateral ligament ruptures occur at the proximal attachment to the middle phalanx. Partial tears can be treated with nonsurgical and conservative methods. Splinting, buddy taping, extension block placement, and temporary fixation with a Kirschner wire would be incorrect.

A 6-month-old boy who sustained a brachial plexus injury during delivery is brought for evaluation. On examination, the left elbow is held in extension and arm is positioned in internal rotation. He has active flexion of the fingers and wrist. Extension of the fingers and wrist is weak; anti-gravity test of the elbow extension discloses weakness. There is no active elbow flexion or shoulder abduction; Horner's sign is absent. Which of the following structures is most likely injured? A) C7 roots B) C8-T1 roots C) Lateral cord D) Posterior cord E) Upper trunk

E The infant has the classic manifestations of an upper trunk (C5-6), or Erb's, palsy (weak or absent elbow flexion, shoulder abduction and external rotation, relatively preserved elbow extension and distal wrist/hand flexion). The "waiter's tip" posture of the affected extremity indicates relative sparing of lower root (C8-T1) function. Isolated injury to the C7 root is uncommon and would primarily affect radial nerve innervated muscles such as the wrist and finger extensors, and forearm pronation. Injury to the posterior cord also effects radial nerve innervated muscles and would not alter biceps or deltoid function as observed in this child. Lateral cord damage would result in weak or absent biceps and pectoral function, but would not diminish deltoid or periscapular muscle activity.

A 36-year-old man is brought to the emergency department 2 hours after being stabbed in the hand with a knife. Physical examination shows a wound over the dorsum of the first web space. Exploration with administration of a local anesthetic agent is planned. Which of the following is the most appropriate landmark for injection of the local anesthetic? A ) A1 pulley B ) Flexor carpi radialis C ) Medial epicondyle D ) Pisiform E ) Radial styloid

E The most appropriate location for injection of local anesthesia is the radial styloid. Wrist blocks are useful for providing anesthesia during hand surgery. Sensation is blocked, while preserving movement of the extrinsic musculature. The sensory supply to the hand is provided by the median, ulnar, and radial nerves. The median nerve innervates the volar radial aspect of the hand, including thumb, index, long, and the radial half of the ring finger. The median nerve can be blocked as it travels between the flexor carpi radialis and palmaris longus tendons. Sensation to the thenar eminence is provided by the palmar cutaneous branch, which divides about 5 to 7 cm proximal to the wrist crease. The ulnar nerve innervates the ulnar aspect of the hand, including the ulnar aspect of the ring finger and the little finger. At the level of the wrist, the ulnar nerve lies radial to the flexor carpi ulnaris tendon, and ulnar to the ulnar artery. The dorsal cutaneous branch of the ulnar nerve branches proximal to the wrist crease and travels dorsally to innervate the ulnar aspect of the dorsum of the hand. It can be blocked as it crosses the region of the ulnar styloid. The radial nerve provides sensation to the dorsal radial aspect of the hand, including the first web space. The superficial radial nerve divides into several branches in the region of the radial styloid and can be blocked by subcutaneous infiltration over the radial styloid proximal to the anatomical snuffbox. This patient has a wound in the dorsal first web space, which is innervated by the radial nerve; hence, injection at the radial styloid level will provide anesthesia to the area. A digital block can be performed with local injection in the region of the A1 pulley but would result in anesthesia too distal to the area of interest in this case. The median nerve can be anesthetized by injecting local anesthetic just ulnar to the flexor carpi radialis tendon. Injection just posterior to the medial epicondyle would result in blockade of the ulnar nerve at the elbow. The ulnar nerve can be blocked at the level of the wrist by injecting radial to the flexor carpi ulnaris tendon and pisiform bone.

A 45-year-old man sustains a laceration of the ulnar nerve proximal to the elbow. He has loss of intrinsic hand function. At the time of surgical repair, there is a 1-cm gap in the nerve. Which of the following is the most appropriate management of this injury? A) Interposition nerve conduit B) Interposition nerve grafting C) Primary repair only D) Primary repair with anterior transposition E) Primary repair with distal nerve transfer

E The most appropriate management of this injury is to repair the ulnar nerve primarily and perform a distal nerve transfer as well. The ulnar nerve is one of the two upper extremity nerves that supply motor input to the intrinsic muscles of the hand (the other being the median nerve). It supplies the interossei, hypothenar muscles, ulnar lumbricals, and the adductor pollicis muscles. These small muscles of the hand are vital for proper thumb and finger function. Because of their small size and delicate nature, they are very sensitive to denervation, and over a period of 9 to 12 months atrophy beyond repair. The aim of nerve repair is to reestablish nerve signals to the end-organ (i.e., muscle) prior to irreversible denervated muscle. The injury described is a good 18 inches or so away from the hand, and regeneration of the motor fibers to the intrinsic muscles would only occur after at a rate of an inch a month, by which time permanent atrophy would have already occurred. Primary repair alone can lead to suboptimal intrinsic muscle function despite excellent technical repairs. Distal nerve transfers help prevent the denervational atrophy by "babysitting" the muscles during the time it takes for the ulnar nerve to regenerate its motor fibers to the end-organ/muscles. The terminal branch of the anterior interosseous nerve (AIN) is most commonly used as the donor nerve. An end-side neurorrhaphy is performed to the ulnar motor fascicles in the distal forearm, a distance which results in reinnervation of the intrinsic muscles well before the 9- to 12-month mark. The known topography of the ulnar nerve allows the surgeon to coapt the donor nerves to the appropriate motor recipient site of the ulnar nerve. An end-end coaptation of the terminal AIN to the motor fascicle of the ulnar nerve is also an option. Nerve grafts or conduits are not required to repair the nerve injury in this clinical scenario. A 1-cm gap can usually be primarily repaired after dissecting the nerve and freeing it up proximally and distally. If further length is needed for tension-free repair, the ulnar nerve can be transposed anteriorly out of its natural position, giving another few centimeters of length.

A 53-year-old woman comes to the office after undergoing fixation of a humerus fracture 17 months ago. Physical examination shows inability to extend the wrist, fingers, and thumb. This has been present since the time of injury, without any recovery of function. Tendon transfers are planned. Transfer of which of the following muscles is most appropriate for restoration of wrist extension? A) Brachioradialis B) Flexor carpi ulnaris C) Flexor digitorum profundus D) Palmaris longus E) Pronator teres

E The most appropriate muscle to transfer for restoration of wrist extension is the pronator teres. The radial nerve can be injured as a result of humerus fracture and/or surgery as it crosses the spiral groove of the humerus. The resultant radial nerve palsy will cause inability to extend the wrist, fingers, and thumb. Reinnervation of the muscle ideally should be completed within 12 to 18 months after injury to allow for recovery. In this patient, who has high radial nerve palsy after humerus fracture, the time following injury has been too long, so nerve repairs or nerve transfers are not a viable option, and tendon transfer is the procedure of choice. Tendon transfer involves the use of a noncritical or expendable donor tendon to provide a missing function. The tendon to be transferred should have adequate strength and range of motion to provide the desired function. Ideally the tendon used should have synergistic action and allow for tenodesis to facilitate reeducation. The pronator teres is a median nerve-innervated muscle that has adequate power and excursion to provide wrist extension. It is typically transferred to the extensor carpi radialis brevis (as opposed to the extensor carpi radialis longus) in these cases to provide for more centrally oriented wrist extension. The brachioradialis is a radial nerve-innervated muscle and will not be functioning in this patient who has a high radial nerve palsy. In low radial nerve palsies, it can be used to restore thumb extension. The brachioradialis can also be used to restore finger or wrist extension, as well as finger or thumb flexion in the appropriate patient. The flexor carpi ulnaris would have adequate power and excursion, but it is not synergistic and it would be difficult to learn to use a wrist flexor to power wrist extension, as it provides an opposite function. This is typically used to restore finger extension, as it would take advantage of the tenodesis effect. The flexor digitorum profundus would be synergistic with wrist extension, but it does not have independent muscle bellies and its use would require sacrifice of important finger flexor activity. The palmaris longus does not have sufficient power to provide for wrist extension. It can be used as a transfer for thumb extension.

A 24-year-old woman comes for evaluation 6 days after sustaining a jamming injury to the long finger of the left hand with resultant central slip disruption and acute boutonnière injury. X-ray studies are negative for fracture or dislocation. Treatment with splint immobilization is planned. Which of the following is the most appropriate position of the finger for application of the splint? A) Distal interphalangeal (DIP) joint extended, proximal interphalangeal (PIP) joint extended B) DIP extended, PIP free C) DIP flexed, PIP flexed D) DIP flexed, PIP free E) DIP free, PIP extended

E The most appropriate position for splint immobilization of an injury leading to a boutonnière deformity is with the distal interphalangeal (DIP) joint free and the proximal interphalangeal (PIP) joint extended. The patient sustained a central slip disruption of the long finger of the left hand, resulting in an acute boutonnière deformity. Patients with boutonnière deformity have flexion at the PIP joint and hyperextension at the DIP joint. There is tearing of the extensor tendon from its insertion at the base of the middle phalanx, resulting in decreased ability to extend the finger at the PIP joint. As a result, the lateral bands fall volar to the axis of rotation at the PIP joint, changing their force from extension to flexion at the PIP joint. The extensor force of the lateral bands is then directed toward the terminal tendon at the DIP joint, resulting in hyperextension of the DIP joint. Splint immobilization of the PIP joint in extension and the DIP joint free to flex permits healing of the central slip back to the middle phalanx. Flexion of the DIP joint through movement will tend to cause the lateral bands to migrate dorsally again, reversing the effect of the boutonnière deformity. When the lateral bands move dorsal to the axis of rotation at the PIP joint, their extensor force is restored and the DIP hyperextension resolves. Splint immobilization of both joints in extension will not encourage the dorsal migration of the lateral bands. Splint immobilization with the DIP joint in extension and the PIP joint free is recommended in zone 1 extensor tendon injuries (i.e., mallet finger).

A 20-year-old man comes to the office 2 months after "jamming" the long finger of the right hand in a rugby game. On examination, the patient has a boutonnière deformity. The distal interphalangeal (DIP) joint has 20 degrees of hyperextension and active flexion to 85 degrees. X-ray study shows no fracture. Which of the following is the most appropriate initial treatment? A) Open central slip repair B) Resection of the lateral bands and oblique retinacular ligament C) Resection of the lateral bands only D) Splinting of the proximal interphalangeal (PIP) and DIP joints in extension E) Splinting of the PIP in extension and active DIP flexion

E The patient has a boutonnière deformity that is passively correctable. In most instances, this can be successfully managed with splinting of the PIP joint in extension while allowing active DIP flexion. Splinting of the PIP in extension helps restore central slip continuity; active DIP flexion with the PIP joint extended draws the tight, volarly displaced lateral bands into a more dorsal position while reducing DIP joint hyperextension. Holding the PIP and DIP joints both in extension will not correct the deformity. Open central slip repair is indicated if there is an open wound, but that is not the case in this scenario. Resection of the distal lateral bands only is a reasonable treatment for chronic deformity by relaxing the lateral bands. There is no role for resection of both lateral bands and oblique retinacular ligament in the management of this condition.

A 35-year-old woman comes for evaluation of a 6-month history of increasing numbness of the right long finger of the dominant hand. She delivered a healthy newborn 6 months ago. She reports that the numbness awakens her from sleep and resolves after she shakes her hand. Physical examination shows a two-point discrimination of 5 mm in all digits. Which of the following is the most likely abnormal electrodiagnostic finding in this patient? A) Motor action potential amplitude of 25 ?V B) Motor latency of 4 ms C) Occasional fasciculations D) Positive sharp waves E) Sensory action potential latency of 4 ms

E The patient has early carpal tunnel syndrome. She only exhibits sensory findings with numbness that resolves. Two-point discrimination is abnormal in late carpal tunnel syndrome when there is irreversible nerve damage. There are no motor symptoms at this point. Electrodiagnostic studies are a two-part examination consisting of sensory action potentials (SAP), also referred to as nerve conduction studies (NCS) and electromyography (EMG). Findings in EMG latency for muscle abnormality are >4.0 ms. Normal muscles show occasional fasciculations with high fibrillations 5 weeks after denervation. Normal motor nerve shows no sharp waves and no fasciculations. Mild motor latencies on NCS/EMG are >4.0 ms. Motor latencies of >6.0 are considered severe. Because this is early carpal tunnel syndrome with no evidence of motor weakness, a motor latency of 4.0 is unlikely. Similarly, a motor action potential of 25 ?V is unlikely. Findings in SAP distal latency for sensory abnormality are >3.5 ms and an amplitude <15 ?V (normal is 15-25 ?V). Therefore, sensory action potential latency of 4.0 ms is correct.

A 50-year-old male construction worker is evaluated for weakness of grip and pinch with inability to touch index finger to thumb. The patient is referred from a neurologist with a diagnosis of anterior interosseous nerve syndrome. Which of the following muscles is most likely to be spared? A) Flexor digitorum profundus to index B) Flexor digitorum profundus to middle C) Flexor pollicis longus D) Pronator quadratus E) Pronator teres

E The pronator teres is innervated by the median nerve prior to its take off of the anterior interosseous nerve (AIN), which is why it cannot be affected by AIN syndrome. Pronation generated by the pronator teres or the pronator quadratus would be indistinguishable clinically. The AIN is a terminal branch off the median nerve that innervates the flexor digitorum profundus to the index and middle fingers, flexor pollicis longus (FPL) and pronator quadratus. The AIN arises from the median nerve approximately 4 to 6 cm distal to the medial epicondyle. It travels between the flexor digitorum profundus (FDP) and flexor digitorum superficialis (FDS) initially, and then between the FPL and FDP. Then it lies on the volar surface of the interosseous membrane and travels with the anterior interosseous artery, terminating in the pronator quadratus and then the wrist joint capsule and the intercarpal, radiocarpal, and radioulnar joints. The nerve originates from C5-T12, becoming the medial and lateral cords of the brachial plexus and then becoming the median nerve.

An 8-year-old boy is brought to the emergency department after sustaining injury to the right upper extremity, 3-cm proximal to the antecubital fossa. Which of the following factors is associated with improved functional outcomes following peripheral nerve repair? A) Fewer suture strands used in the nerve repair B) Higher-tension nerve repair C) Increasing time between nerve injury and repair D) More proximal nerve injury E) Younger patient age

E The repair of peripheral nerve injuries can be affected by several factors. Younger patients tend to have improved outcomes compared with older patients. Although there is no consensus on the optimal timing for nerve repair, earlier repairs have been shown to have better outcomes than those attempted at later time points. The level at which the injury has occurred can also affect the outcome. The more proximal the injury, the worse the prognosis in terms of motor and sensory return. Moreover, more complete and rapid regain of function occurs in more proximally innervated muscles. Finally, technical aspects of the nerve repair can also affect outcomes. Minimal tension and an increasing number of suture strands crossing the repair site are both associated with improved function.

A 25-year-old woman comes to the office with a 2-day history of difficulty moving the left thumb. Eight weeks ago, she sustained a nondisplaced distal radius fracture. She has been out of a cast for the past 2 weeks. On examination, thumb retropulsion is absent. Which of the following is the most appropriate definitive treatment? A) Fusion of the carpometacarpal (CMC) joint of the thumb B) Fusion of the interphalangeal (IP) joint of the thumb C) Transfer of the anterior interosseous nerve to the recurrent branch of the median nerve D) Transfer of the extensor indicis proprius (EIP) tendon to the abductor pollicis brevis tendon E) Transfer of the EIP tendon to the extensor pollicis longus tendon

E The scenario depicts a classic case of extensor pollicis longus (EPL) tendon rupture following distal radius fracture. The reported incidence of EPL tendon rupture ranges from 0.2 to 3%. Ruptures can occur after internal or external fixation due to impingement of hardware on the tendon or due to ischemic changes in the tendon due to swelling of the tendon and the third dorsal compartment. Reconstruction of the EPL tendon can be accomplished either by tendon transplantation, typically the palmaris interposition between the proximal and distal healthy segments of the EPL tendon, or by transfer of the EIP to the distal segment of EPL tendon. When identified and treated before the EPL muscles retract and shorten, equivalent outcomes can be achieved. Later treatment necessitates tendon transfer. Fusion of the thumb IP joint may be useful in flexor pollicis longus ruptures that cannot be repaired, but this would not restore thumb retropulsion. Fusion of the CMC joint can alleviate pain from basal joint arthritis, but would result in further loss of motion of the thumb. Transfer of the EIP to the abductor pollicis brevis and transfer of the anterior interosseous nerve to the recurrent branch of the median nerve are techniques for restoring thumb palmar abduction/opposition and would not restore retropulsion/extension.

A 25-year-old woman comes for evaluation because she has difficulty gripping things with her dominant right hand. She sustained a stab wound to the left forearm 6 months ago. Physical examination shows a Tinel sign present over the ulnar aspect of the mid volar forearm with a well-healed laceration. A neuroma in-continuity is identified intraoperatively. The neuroma is resected, and a 3-cm nerve gap remains. Which of the following is the most appropriate management? A ) Bridge the gap with a synthetic nerve conduit B ) Dissect the nerve both proximally and distally with an end-to-end neurorrhaphy C ) Proceed to tendon transfers to assist with grip D ) Reconstruct the nerve defect with a lateral antebrachial cutaneous nerve graft E ) Sural nerve cable grafting

E The sural nerve is a common source of nerve graft material. It is formed from the medial cutaneous sural nerve that originates from the tibial nerve. Synthetic nerve conduit repair of nerves has not proven effective in gaps larger than 2 cm and is less effective in motor nerve defects. A tension-free repair is the goal for the nerve anastomoses. When an acute laceration occurs, primary end-to-end neurorrhaphy is indicated. When the treatment is delayed and a neuroma exists, the type of repair is dictated by the size of the nerve gap. Nerve grafting is indicated to bridge a defect when more than 10% elongation of the nerve would be necessary to bridge the gap. Tendon transfers would not be indicated at this point. The lateral antebrachial cutaneous nerve is the distal continuation of the musculocutaneous nerve. Given the size of the defect and the need for at least two cable grafts, this nerve would not be appropriate.

A 30-year-old woman comes to the office because of pain and swelling of the right radial/distal forearm. The pain worsens with ulnar deviation of the wrist over a clasped thumb. Immobilization and a corticosteroid injection have failed, and surgical release is planned. Which of the following anatomic findings is most likely to have contributed to this patient's condition? A) Absent extensor pollicis brevis B) Combined first and second compartments C) Distal muscle belly on extensor pollicis brevis tendon D) Multiple slips of abductor pollicis longus E) Septum between abductor pollicis longus and extensor pollicis brevis

E There is considerable variability in the anatomy of the first dorsal compartment of the wrist; this is particularly true when considering the presence or absence of a septum in the compartment. The significance of these variations is that they may predict failure of steroid injections to resolve de Quervain disease and must be considered when releasing the compartment surgically to ensure that a complete release of all tendons is performed. While studies vary, based on methodology of imaging, cadaveric dissection, or surgical findings, there is consistency that septa are more common in patients presenting with de Quervain disease than in the general population. Studies have shown no association between the number of APL slips and de Quervain disease.

A 28-year-old man is brought to the emergency room because of pain, swelling, and deformity of the right small finger and a laceration over the finger after being involved in a motor vehicle collision. Physical examination shows a 7-mm laceration over the fifth metacarpal shaft. X-ray studies confirm a significantly displaced transverse metacarpal shaft fracture of the right small finger. Which of the following is the most appropriate management? A) Closed reduction of the fracture and splint immobilization followed by 7 to 10 days of outpatient oral antibiotic therapy B) Closed reduction of the fracture followed by splint immobilization only C) Emergent debridement in the operating room followed by external fixation of the fracture D) Inpatient intravenous antibiotic therapy and debridement in the operating room within 48 hours E) Irrigation of the wound in the emergency room, splint immobilization, 24 hours of oral antibiotic therapy, and elective operative fixation of the fracture

E This patient has a displaced Grade I open fracture of the fifth metacarpal shaft. Appropriate management includes irrigation and debridement of the wound in the emergency room, splint immobilization, and a brief course of antibiotic therapy. This allows the fracture to be managed as if it were a closed injury. Transverse shaft fractures typically are unstable and require some type of elective fixation beyond splint immobilization. Open fractures usually are classified according to the Gustilo-Anderson scale, although outcomes of upper extremity open fractures do not necessarily correlate to this system. Grade I injuries are defined as fractures with open wounds <1 cm. Grade II fractures have open wounds measuring between 1 and 10 cm. Grade IIIA fractures have wounds >10 cm with comminution but adequate soft-tissue coverage. Grade IIIB fractures have wounds >10 cm with extensive periosteal stripping that requires soft-tissue reconstruction. Grade IIIC fractures have large wounds with associated vascular injury. In upper extremity fractures, studies have not correlated time to operative debridement or the administration of antibiotic therapy with ultimate outcomes in terms of infection, malunion, or osteomyelitis. Factors that have been shown to correlate to poor outcomes in large-scale studies have been high injury severity score, Gustilo grade III, and fractures of the tibia/fibula. Shorter duration of antibiotic administration for open fractures has not been shown to result in a higher rate of infection. The current recommendation from the American Society of Orthopedic surgeons is a 3-day regimen of antibiotics for Grade I/II injuries and 5 days for Grade III. Grade I injuries have only a minor soft-tissue component and there is no role for emergent surgery, hospital admission, or the need for provisional external fixation. Closed reduction and splint immobilization are not appropriate for unstable fractures as definitive management in a healthy, young patient. In addition, there is no role for prolonged courses of antibiotic therapy. The fracture can be treated with percutaneous pin fixation or plate fixation depending on the preference of the surgeon.

A 5-year-old boy who underwent closed reduction and pinning of a supracondylar humerus fracture is evaluated in the postanesthesia care unit because of absence of palpable radial pulse in the left wrist. Physical examination shows the left hand is warm with color similar to the right hand. No pulses are palpable at the level of the wrist. Without surgical exploration, which of the following is the most likely outcome for the left forearm? A) Fibrosis of the deep flexor compartment B) Ischemic necrosis of the radial hand C) Median nerve palsy D) Physeal arrest in the forearm bones E) No functional impairment

E Vascular impairment related to pediatric supracondylar fractures occurs in roughly 5 to 10% of patients. Given the nontrivial morbidity of exploration of the brachial artery after reduction of the supracondylar fracture, controversy exists over what criteria should prompt surgical intervention. Recent large, retrospective studies have demonstrated that absence of pulses in the wrist after reduction is not, by itself, associated with poor outcomes. Most patients who lack pulses immediately after reduction, but who have an otherwise perfused hand, go on to recover a palpable radial pulse within 24 to 48 hours after the reduction. In this patient with a perfused hand, observation can be expected to result in a good outcome. Physeal arrest can occur secondary to ischemia, but this would not be the expected outcome based on the perfusion of the hand. Fibrosis of the deep flexor compartment (Volkmann ischemic contracture) is associated with compartment syndromes but would be rare in this setting. Median nerve impairment associated with the vascular insufficiency at the time of injury is advocated as an indication for surgical exploration. Development of a new median palsy as a late result of the vascular injury is unlikely. Rare case reports of late development of ischemia in the forearm and hand occur, and families should be counseled to watch for color or temperature changes in the hand following discharge after the recommended observation period of 24 to 48 hours.

A 50-year-old woman has significant tenderness of the residual tip of the index finger 8 weeks after undergoing amputation of the fingertip. Physical examination shows significant stump tenderness. Which of the following therapy modalities is most appropriate for desensitization of the amputation stump? A ) Functional electric stimulation B ) Immobilization C ) Kinesiology tape D ) Semmes-Weinstein monofilaments E ) Vibration

E Vibration is a modality that may be used in therapy to desensitize an amputation stump neuroma. The treatment consists of vibratory stimulation applied to the periphery of the sensitive area and then gradually moving toward the center. Some additional methods used in therapy to treat neuroma pain include desensitization, massage, and transcutaneous nerve stimulation. Functional electric stimulation is similarly not a therapeutic modality. Kinesiology taping is used for edema control and comfort. It would not be used on an amputated stump. Semmes-Weinstein monofilaments are used to quantify sensation to fine touch and are not used for desensitization.


Set pelajaran terkait

Abnormal Psychology Exam 3 part 3

View Set

Chapter 1: What is deep learning

View Set

Real Estate Practice Chapters 1 - 15 (Exam Prep Text)

View Set

chapter 4 entrepreneurshipWhich is a reason why managers develop company objectives?

View Set

Chapter 39: Equal Employment Opportunity Law

View Set

Chapter 9 Lesson 1 Colonists Protest British Rule

View Set

😲😲APWH STUDY GUIDE!!!!! 😲😲

View Set

MasteringA&P CH08 IHW Biol 1020 (2018)

View Set